You are on page 1of 90

Item��1

GLP-
1��receptor��agonists��are��the��most��likely��of��these��medications��to��lead��to
��weight��loss.��They��work��by
activating��the��GLP-
1��receptors,��which��increases��insulin��secretion,��decreases��glucagon��secretio
n,��slows
gastric��emptying,��and��increases��satiety.��Other��diabetes��medications��that��m
ay��promote��weight��loss��include
metformin,��amylin��mimetics,��and��SGLT-
2��inhibitors.��The��other��medications��listed��have��all��been��shown��to
cause��weight��gain.��DPP-4��inhibitors��are��felt��to��be��weight��neutral.

Item��2

The��hepatitis��C��virus��is��spread��through��contact��with��infected��blood��(usu
ally��unscreened��blood��transfusions,
which��were��the��norm��before��1992),��intravenous��drug��use,��or��high-
risk��sexual��activity.��However,��studies
have��shown��that��screening��only��patients��with��high-
risk��medical��or��clinical��histories��will��identify��just
20%-
33%��of��infected��patients.��As��a��different��strategy,��both��the��CDC��and��the
��U.S.��Preventive��Services
Task��Force��have��recommended��one-
time��screening��of�patients��born��between��1945��and��1965.��It��is��estimated
that��this��cohort��includes��75%��of��all��patients��who��have��chronic��hepatitis
��C.

Item��3

Family��physicians��are��frequently��consulted��by��patients��and��institutions��re
garding��outbreaks��of��head��lice.
Optimal��treatment��involves��home��eradication��measures��and��medication.��Permet
hrin��1��%��shampoo��remains
the��first-
line��treatment.��Ivermectin��appears��effective��for��treatment��but��is��not��FDA
��approved��and��is��not
considered��first-
line��treatment.��Lindane��carries��an��increased��risk��of��toxicity��and��should�
�not��be��used��in
children��weighing��<��50��kg.��Topical��benzyl��alcohol��is��FDA-
approved��for��treatment��but��is��expensive.��The
current��recommendation��from��the��American��Academy��of��Pediatrics��Council��on�
�School��Health��and
Committee��on��Infectious��Diseases��is��to��return��children��to��class��despite��
the��presence��of��nits.
1

Item��4

This��patient��most��likely��has��ischemic��colitis,��given��his��abdominal��pain,�
�bloody��diarrhea,��and��cardiovascular
risk��factors.��Peptic��ulcer��disease��is��unlikely��because��the��nasogastric��as
pirate��was��negative.��Diverticular
bleeding��and��angiodysplasia��are��painless.��Infectious��colitis��is��associated�
�with��fever.

Item��5

Overdiagnosis��is��the��diagnosis��of��a��disease��that��will��not��produce��sympto
ms��during��a��patient�s��lifetime.��It
tends��to��occur��with��cancers��that��have��very��slow��rates��of��growth.��Prosta
te��cancer��is��most��often��a
slow-
growing��cancer��and��is��often��present��without��symptoms��in��older��men.��The��
introduction��of
prostate-
specific��antigen��(PSA)��screening��was��accompanied��by��a��marked��rise��in��the
��rate��of��diagnosis��of
prostate��cancer��while��mortality��decreased��much��less��significantly,��and��thi
s��decrease��was��probably��largely
attributable��to��improved��treatment.��The��problem��of��overdiagnosis��remains��a
��significant��problem��with��PSA
screening��and��was��a��factor��in��the��U.S.��Preventive��Services��Task��Force�s�
�recommendation��against��routine
PSA-based��screening��for��prostate��cancer��(D��recommendation).

Item��6

This��patient��most��likely��has��patellofemoral��pain��syndrome,��as��evidenced��b
y��her��recent��increased��activity
and��anterior��knee��pain��with��increased��lateral��patellar��tracking.��The��most
��effective��treatment��for
patellofemoral��pain��syndrome��is��physical��therapy��and��activity��modification.
��Glucosamine��and��chondroitin
have��been��used��to��treat��osteoarthritis��in��the��past��but��should��not��be��u
sed��to��treat��symptomatic��osteoarthritis
of��the��knee,��according��to��the��American��Academy��of��Orthopedic��Surgeons.��I
ntra-articular��joint��injections
can��be��used��for��knee��arthritis��but��are��not��indicated��for��patellofemoral�
�pain��syndrome.��Knee��arthroscopy
would��not��be��indicated��for��patellofemoral��syndrome��without��evidence��of��in
ternal��knee��derangement.��The
evidence��for��patellar��taping��is��inconsistent,��but��it��may��be��helpful��as��
an��adjunct��treatment.

2
Item��7

In��general,��all��patients��with��hypertension,��regardless��of��age,��benefit��fr
om��treatment.��Patients��>80��years��of
age��who��are��healthy��and��functionally��independent��should��be��treated��accord
ing��to��current��recommendations
for��patients��>��65��years��old.��Available��guidelines��recommend��a��target��blo
od��pressure��of��<��150/90��mm��Hg
unless��the��patient��is��frail��or��has��numerous��comorbidities.��In��these��case
s,��the��target��blood��pressure��should
be��determined��on��a��case-by-case��basis��(SOR��B).

Item��8

The��likelihood��of�azithromycin��inducing��an��arrhythmia��is��small,��but��given�
�its��widespread��use��this��possibility
must��be��kept��in��mind.��The��arrhythmia��results��from��prolongation��of��the��Q
T��interval��and��is��also��more
common��in��patients��with��a��prior��cardiac��history.��The��other��three��antibio
tics��have��not��been��implicated��in��an
increase��in��cardiac��deaths.

Item��9

This��child��most��likely��has��transient��synovitis,��based��on��the��fact��that��
he��is��well��appearing��and��afebrile,��and
his��WBC��count,��erythrocyte��sedimentation��rate,��C-
reactive��protein��level,��and��radiographs��are��all��normal.
This��is��a��self-
limited��inflammatory��condition,��and��he��will��likely��respond��to��NSAIDs��such
��as��ibuprofen��and
relative��rest.��Physical��therapy��and��orthopedic��consultation��are��not��requir
ed,��as��this��condition��lasts��less��than
a��week.��Septic��arthritis��would��require��treatment��with��intravenous��antibiot
ics,��but��there��is��no��evidence��that
he��has��this��condition.

Item��10
Calcium��channel��blockers��such��as��verapamil��or��diltiazem��can��be��used��acut
ely��to��decrease��heart��rate��and
terminate��supraventricular��tachycardia��(SVT)��and��chronically��to��prevent��SVT
��recurrence.��Adenosine,
amiodarone,��esmolol,��and��procainamide��all��can��be��used��to��treat��SVT��acute
ly,��but��they��are��not��suitable��for
long-term��therapy.

Item��11

This��presentation��is��consistent��with��P-
thalassemia��minor��trait��in��a��generally��asymptomatic��patient.
Hemoglobin��electrophoresis��will��be��abnormal,��with��HbA2��increased��and��HbA��
decreased.��The��free��T4��level
is��likely��to��be��normal��in��a��patient��with��a��normal��TSH��level.��A��ferrit
in��level��is��also��likely��to��be��normal��given
the��normal��levels��of��hemoglobin��and��hematocrit.��A��fluorescent��spot��test��
is��used��to��screen��for��G-6-PD
deficiency,��which��would��be��associated��with��bite��cells��and��Heinz��bodies.��
A��vitamin��B12��level��would��be��useful
for��evaluating��macrocytosis,��which��is��not��present��in��this��patient.

Item��12

A��diagnosis��of�delirium�based��on��the��Confusion��Assessment��Method��(CAM)��alg
orithm��requires��the��presence
of��an��acute��onset��and��a��fluctuating��course,��inattention,��and��either��an��
altered��level��of��consciousness��or
disorganized��thinking.��The��patient��described��in��this��question��exhibits��an�
�acute��onset,��fluctuation,��inattention,
and��an��altered��level��of��consciousness.

This��patient�s��presentation��is��more��consistent��with��delirium��than��encephal
itis,��as��patients��with��encephalitis
frequently��have��signs��of��systemic��illness��such��as��fever,��lethargy,��seizur
es,��and��neurologic��deficits,��as��well
as��a��nonspecific��rash��in��some��cases.��Furthermore,��the��fluctuations��in��th
e��level��of��consciousness��seen��in
delirium��do��not��occur��with��encephalitis.��Vascular��dementia��and��Alzheimer�s
��disease��develop��over��years,
not��acutely��as��in��this��case.��Stroke,��while��a��consideration��and��a��potent
ial��cause��of��delirium,��would��not��be
the��most��likely��diagnosis��in��an��older��patient��hospitalized��with��pneumonia
.

Item��13
Amyotrophic��lateral��sclerosis��is��a��pure��motor��disorder��involving��anterior�
�horn��cells��in��the��spinal��cord��and
their��brainstem��homologues.��It��may��begin��as��lower��motor��or��bulbar��palsy,
��and��the��median��survival��is��3-5
years.��Pseudoclaudication��is��associated��with��spinal��stenosis,��and��paresthes
ias��are��caused��by��sensory
problems.��A��bull�s-
eye��rash��is��associated��with��Lyme��disease.��Charcot��joint��is��seen��with��co
nditions��that
involve��sensory��loss,��such��as��diabetes��mellitus��or��syphilis.

Item��14

Causes��of��pleuritic��chest��pain��include��pneumonia,��chest��wall��trauma,��pulm
onary��embolus,��and��vasculitis.
If��these��conditions��are��deemed��unlikely��based��on��the��history,��physical��e
xamination,��and��limited��laboratory
studies,��a��chest��radiograph��is��obtained.��If�this��is��within��normal��limits�
�then��viral��pleuritic��pain��is��most��likely,
and��can��be��treated��with��an��NSAID.��Given��that��the��history��and��physical��
findings��are��not��suspicious��for
thromboembolism��and��a��D-
dimer��is��negative,��anticoagulation��is��inappropriate.��With��no��other��systemi
c
symptoms��or��findings��of��collagen-
vascular��disease,��corticosteroids��are��not��indicated.��Since��there��is��no��ri
b
tenderness��and��no��radiographic��findings��of��an��acute��rib��fracture,��a��rib�
�belt��is��not��indicated.

Item��15

Methadone��has��a��widely��variable��half-life��(7-
72��hours)��and��bioavailability,��as��well��as��an��inactive��sedating
metabolite.��It��is��metabolized��and��cleared��by��the��liver,��and��should��there
fore��not��be��used��in��patients��with
severe��hepatic��impairment.��It��is,��however,��a��reasonable��option��for��patien
ts��with��severe��renal��impairment
(SOR��C).��Morphine,��oxycodone,��fentanyl,��and��hydromorphone��can��be��used��cau
tiously��in��patients��with
hepatic��impairment,��with��reductions��in��the��initial��dosage��and��close��monit
oring��(SOR��C).

Item��16
This��patient��is��obese��and��has��an��isolated��elevation��of�ALT.��The��initial�
�history��and��workup,��including��testing
for��chronic��hepatitis,��a��medication��review,��and��alcohol��screening,��have��a
ll��been��conducted.��Current
guidelines��suggest��iron��studies��to��screen��for��hemochromatosis��as��part��of�
�the��initial��evaluation��(SOR��C).
Anti-smooth-
muscle��antibody��and��ceruloplasmin��levels��can��detect��more��rare��causes��of��
hepatocyte
inflammation��(autoimmune��hepatitis��and��copper��deposition��from��Wilson�s��dise
ase).��These��tests��can��be
considered��in��persistent��transaminitis��based��on��clinical��judgment��(SOR��C).
��Hepatitis��D��is��a��superinfection
seen��in��patients��with��hepatitis��B,��and��is��not��a��consideration��in��this��
patient.��A��liver��biopsy��may��also��be
considered��in��persistent��transaminitis,��especially��in��the��setting��of�clinic
al��or��serologic��evidence��of�advanced
liver��fibrosis,��but��is��not��recommended��at��this��stage��(SOR��C).��Patients��
with��fatty��liver��disease��are��at
increased��risk��for��developing��liver��cirrhosis.

Item��17

Cats��that��are��infected��with��Toxoplasma�gondii�cm��shed��the��virus��for��weeks
��when��newly��infected.��Cleaning
the��cat�s��litter��box��is��a��high-
risk��activity,��and��transmission��to��the��fetus��can��occur.��Toxocara��cat/��in
fection
is��less��common��and��usually��results��from��ingesting��contaminated��cat��meat��
or��soil��containing��cat��feces.
Chagas��disease��is��spread��by��insect��bites.��Cysticercosis��is��generally��acqu
ired��by��eating��contaminated��pork.
Aspergillosis��usually��occurs��in��immunocompromised��people��who��inhale��the��sp
ores��found��in��soil��(SOR��A).

Item��18

A��review��of�multiple��studies��has��revealed��that��continuous��daily��macrolide�
�antibiotic��use��for��1��year��decreased
the��number��of��exacerbations��in��older��patients��with��COPD��(SOR��A).��Quality
��of��life��did��not��show
improvement,��however,��and��resistance��to��macrolide��and��quinolone��antibiotics
��rose.��There��was��no��increase
in��conduction��abnormalities��or��tinnitus,��but��the��incidence��of��hearing��los
s��increased��with��the��use��of
azithromycin.

6
Item��19
Patients��with��new��health��insurance��plans��or��policies��purchased��under��the�
�Patient��Protection��and��Affordable
Care��Act��are��expected��to��be��covered��for��certain��services.��Preventive��ser
vices��with��a��U.S.��Preventive
Services��Task��Force��grade��of�A��or��B,��such��as��colorectal��cancer��screening
,��are��covered��without��cost-sharing
when��performed��by��an��in-
network��provider.��Although��coverage��for��vaccines��such��as��diphtheria,��tetan
us,
and��influenza��is��required��with��these��plans,��those��that��are��needed��only��
for��travel��are��not��covered.��In��addition,
although��consumers��can��purchase��vision��and��dental��coverage��in��addition��to
��health��coverage��through��the
marketplace,��they��are��not��a��required��part��of��coverage.��While��coverage��fo
r��obesity��counseling��is��required,
coverage��for��bariatric��surgery��still��varies��by��state.

Item��20

The��transjugular��intrahepatic��portosystemic��shunt��(TIPS)��procedure,��by��shun
ting��blood��destined��for��the��liver
into��the��systemic��circulation,��lowers��pressure��in��the��portal��veins,��there
by��decreasing��portal��system
hypertension��and��making��variceal��bleeding��and��portal��hypertensive��gastropat
hy��less��likely.��TIPS��may
decrease��the��likelihood��of��variceal��bleeding��by��as��much��as��90%.��TIPS��al
so��reduces��the��pressure��that��leads
to��ascites��and��lower��extremity��edema,��or��the��massive��edema��of��anasarca.�
�The��diversion��of��blood��from��the
liver��circulation��compromises��the��liver�s��role��in��removing��toxins,��includi
ng��the��serum��marker��ammonia.
This��may��make��patients��more��vulnerable��to��episodes��of��hepatic��encephalopa
thy.

Item��21

In��elderly��patients,��detrusor��instability��is��the��most��common��cause��of��ur
inary��incontinence��in��both��men��and
women.��Incontinence��may��actually��become��worse��after��surgical��relief�of�obst
ructive��prostatic��hypertrophy.

Infection��is��unlikely��to��be��the��cause��of��persistent��incontinence��in��this
��patient��in��the��absence��of��fever��or
symptoms��of��urinary��tract��infection.��Overflow��is��unlikely��in��the��absence�
�of��residual��urine.��Impaction��is��a
relatively��rare��cause��of��urinary��incontinence,��and��associated��findings��wou
ld��be��present��on��the��rectal
examination.��Normalization��of��the��urinary��stream��and��the��absence��of��resid
ual��urine��reduce��the��likelihood
of��recurrent��obstruction.��The��prostate��would��be��expected��to��remain��enlarg
ed��on��rectal��examination��after
transurethral��resection��of��the��prostate��(TURP).

7
Item��22

Individuals��with��impaired��fasting��glucose��or��impaired��glucose��tolerance��ha
ve��a��higher��risk��of��developing
diabetes��mellitus��in��the��future.��Randomized,��controlled��trials��have��shown�
�that��taking��metformin,
a-
glucosidase��inhibitors��(acarbose),��orlistat,��or��thiazolidinediones��significan
tly��reduces��the��risk��of
developing��diabetes��mellitus.��The��U.�S.��Diabetes��Prevention��Program��Outcome
s��Study��demonstrated��a��34��%
reduction��in��the��development��of��diabetes��mellitus��at��10��years.��In��additi
on��to��medications,��it��is��also
recommended��that��patients��be��counseled��about��weight��loss��and��engaging��in�
�moderate��physical��activity��for
at��least��150��minutes��per��week.��The��other��medications��listed��are��indicate
d��for��the��treatment��of��diabetes��but
have��not��been��shown��to��be��effective��for��prevention.

Item��23

Desloratadine,��fexofenadine,��levocetirizine,��and��loratadine��are��among��the��s
econd-generation��antihistamines
that��have��a��better��adverse��effect��profile��and��cause��less��sedation��than��
first-generation��antihistamines��(SOR
A).��Second-
generation��antihistamines��have��more��complex��chemical��structures��that��decrea
se��their��movement
across��the��blood-
brain��barrier,��reducing��central��nervous��system��adverse��effects��such��as��se
dation.��Although
cetirizine��is��generally��classified��as��a��second-
generation��antihistamine��and��a��more��potent��histamine
antagonist,��it��is��known��to��cause��sedation��(SOR��A).

Item��24

Trismus��is��almost��universally��present��with��peritonsillar��abscess,��while��vo
ice��changes,��otalgia,��and
odynophagia��may��or��may��not��be��present.��Pharyngotonsillitis��and��peritonsill
ar��cellulitis��may��also��be
associated��with��these��complaints.
Otalgia��is��common��with��peritonsillar��abscess,��otitis��media,
temporomandibular��joint��disorders,��and��a��variety��of��other��conditions.��Peri
tonsillar��abscess��is��rarely��found
in��patients��who��do��not��have��at��least��a��3-
day��history��of��progressive��sore��throat.

8
Item��25

There��are��estimated��to��be��1.6-3.8��million��sports-
related��concussions��each��year��in��the��United��States,��and
appropriate��management��can��prevent��many��of��the��long-
term��complications.��In��this��scenario,��the��patient
meets��the��criteria��for��a��concussion,��with��headache,��nausea,��and��imbalance
��following��an��injury.��She��was
appropriately��removed��from��the��game.��The��next��step��in��management��is��comp
lete��physical��and��cognitive��rest
until��her��symptoms��resolve��without��medication��(SOR��C).��Complete��physical��
rest��includes��avoidance��of��any
physical��activity��that��exacerbates��symptoms.��Cognitive��rest��means��avoidance
��of��activities��that��require
concentration��or��attention,��such��as��schoolwork��and��electronics��use.

Once��the��patient�s��symptoms��have��resolved��without��medication,��she��should��
be��reevaluated��and��can��start��a
graded��return-to-
play��protocol��(SOR��C).��Reevaluation��may��include��repeat��neurocognitive��test
ing��and
postural��stability��testing��(SOR��C).��Both��types��of��testing��can��be��importa
nt��in��the��evaluation��but��there��is
insufficient��evidence��that��either��affects��outcomes.

In��the��initial��evaluation��of��concussion,��imaging��is��not��indicated��in��the
��absence��of��focal��neurologic��findings,
loss��of��consciousness��for��more��than��60��seconds,��or��evidence��of��skull��fr
acture��(SOR��C).��If��imaging��criteria
are��met,��CT��of��the��head��is��indicated.��Plain��films��are��not��indicated��in
��the��evaluation.

There��is��no��evidence��that��protective��gear��reduces��the��incidence��of��concu
ssion��(SOR��C).��Even��though��some
protective��gear��can��prevent��other��types��of�injuries,��it��is��not��indicated�
�for��concussion��prevention.��Eliminating
dangerous��behaviors��such��as��heading��the��ball��in��soccer��can��help,��however
.

Item��26

The��2013��U.S.��Preventive��Services��Task��Force��guidelines��recommend��annual��
lung��cancer��screening��with
low-dose��CT��for��asymptomatic��adults��55-80��years��of��age��who��have��a��30-
pack-year��smoking��history��and
currently��smoke��or��have��quit��smoking��within��the��past��15��years��(grade��B�
�recommendation).��Screening��should
be��discontinued��when��the��patient��has��not��smoked��for��15��years��or��develop
s��a��health��problem��that
substantially��limits��life��expectancy��or��the��ability��or��willingness��to��hav
e��curative��lung��surgery.
9

Item��27

The��fetal��fibronectin��test��is��used��to��help��determine��the��risk��of��early�
�delivery.��Unfortunately,��the��positive
predictive��value��for��delivery��within��the��next��week��is��<20%��in��patients��
who��have��clinical��indications��of
preterm��labor.��In��a��patient��who��does��not��have��any��features��of�preterm��l
abor,��it��does��not��affect��management.
A��negative��result��is��reassuring,��on��the��other��hand,��because��the��negative
��predictive��value��is��>95%.��In
studies��that��tested��gravidas��every��week,��positive��results��had��a��mild��cor
relation��with��preterm��delivery,��but
not��enough��to��guide��management.��False-
positive��test��results��can��be��caused��by��maternal��blood��or��amniotic
fluid,��but��fetal��fibronectin��does��not��indicate��rupture��of��the��membranes��
when��other��tests��are��negative.

Item��28

Fifth��disease,��caused��by��parvovirus��B19,��is��a��common��childhood��infection�
�that��typically��presents��with��a
prodrome��of��low-
grade��fever,��malaise,��sore��throat,��nausea,��and��headache,��followed��a��few��
days��later��by��an
erythematous��facial��rash��on��the��cheeks.��The��rash��fades��in��a��few��days��b
ut��a��lacy,��reticular��rash��develops��on
the��extremities.��Once��the��rash��appears,��patients��are��no��longer��considered
��contagious.��NSAIDs��and
antihistamines��are��often��used��for��symptom��relief��(SOR��A).

Item��29

Idarucizumab��has��been��approved��for��the��urgent��reversal��of��the��anticoagula
nt��effect��of��dabigatran.��The��dose
is��two��consecutive��2.5-
g��infusions��and��the��cost��is��$3500.��Apixaban,��edoxaban,��and��rivaroxaban�al
l��factor
Xa��inhibitors�are��not��affected��by��this��medication��and��they��do��not��curren
tly��have��a��reversal��agent.
10

Item��30

The��American��Academy��of�Family��Physicians��and��the��American��College��of�Obst
etricians��and��Gynecologists
collaborated��on��the��Choosing��Wisely��campaign��recommendation��regarding��sched
uled��inductions��of��labor
or��cesarean��deliveries.��These��organizations��noted��that��delivery��prior��to��
39��weeks,��0��days��is��associated��with
an��increased��risk��of��learning��disabilities��and��may��increase��morbidity��and
��mortality.��While��delivery��prior
to��39��weeks��and��0��days��may��be��indicated��in��certain��situations,��a��matur
e��fetal��lung��test��by��itself��is��not��an
indication��for��delivery��(SOR��C).

Item��31

Benzodiazepines,��such��as��chlordiazepoxide,��can�prevent��alcohol��withdrawal��se
izures.��Anticonvulsants��such
as��carbamazepine,��gabapentin,��and��phenytoin��have��less��abuse��potential��than
��benzodiazepines��but��do��not
prevent��seizures.��Clonidine,��an��a-
adrenergic��agonist,��reduces��the��adrenergic��symptoms��associated��with
withdrawal��but��does��not��prevent��seizures.

Item��32

This��patient�s��pulmonary��function��test��(PFT)��findings��are��consistent��with�
�a��restrictive��defect��with��a��low
DLCO.��Idiopathic��pulmonary��fibrosis,��asbestosis,��hypersensitivity��pneumonitis
,��and��sarcoidosis��are
restrictive��lung��diseases��with��a��low��diffusion��capacity��due��to��alveolar��
damage.��COPD��and��asthma��are��both
obstructive��lung��diseases,��but��can��be��associated��with��an��abnormal��DLCO.��
Morbid��obesity��causes��a
restrictive��lung��disease��associated��with��a��normal��DLCO.��The��presence��of��
chronic��pulmonary��emboli��is
associated��with��a��low��DLCO��due��to��pulmonary��vascular��disease,��but��PFTs��
are��normal.
11

Item��33

An��intranasal��corticosteroid��alone��should��be��the��initial��treatment��for��al
lergic��rhinitis��with��symptoms
affecting��quality��of��life��(SOR��A).��Intranasal��corticosteroids��act��by��decr
easing��the��influx��of��inflammatory
cells��and��inhibiting��the��release��of�cytokines,��thereby��reducing��inflammatio
n��of�the��nasal��mucosa.��Intranasal
corticosteroids��are��more��effective��than��oral��and��intranasal��antihistamines�
�in��the��treatment��of��persistent��or
more��severe��allergic��rhinitis��(SOR��A).��Intranasal��antihistamines��also��have
��more��adverse��effects��than
intranasal��corticosteroids��(SOR��C).��Subcutaneous��and��sublingual��immunotherap
y��are��not��considered
first-
line��treatments��but��should��be��considered��for��moderate��or��severe��persisten
t��allergic��rhinitis��that��is��not
responsive��to��usual��treatments��(SOR��A).

Item��34

Acute��pericarditis��is��a��common��cause��of��chest��pain��in��young��adults.��It�
�presents��with��an��acute��to��subacute
onset��of��severe��retrosternal��and��left��precordial��chest��pain��that��is��char
acteristically��alleviated��by��leaning
forward��or��sitting��up��and��worsened��with��lying��supine.��Viral��and��idiopath
ic��acute��pericarditis��are��most
common��and��occur��at��all��ages,��but��are��more��common��in��young��adults.��The
��pain,��which��is��inflammatory��in
origin,��typically��responds��to��an��NSAID��such��as��indomethacin.��High-
dose��aspirin��(2-4��g/day)��has��also��been
shown��to��be��effective.��Patients��with��pain��resulting��from��myocardial��ische
mia��would��benefit��from
nitroglycerin,��while��those��with��gastroesophageal��reflux��disease��would��benef
it��from��a��proton��pump��inhibitor
such��as��omeprazole.��Hydrocodone��can��be��used��to��relieve��moderate��to��sever
e��pain��but��lacks
anti-inflammatory��properties��(SOR��C).

Ref:

Snyder��MJ,��Bepko��J,��White��M:��Acute��pericarditis:��Diagnosis��and��management
.��Am��Fam��Physician��2014;89(7):553-560.
2)��Kasper��DL,��Fauci��AS,��Hauser��SL,��et��al��(eds):��Harrison's�Principles��of
Internal�Medicine,��ed��19.��McGraw-Hill,��2015,
pp��1571-1573.

Item��35

The��differential��diagnosis��of��acute��nonlocalized��abdominal��pain��is��broad.�
�CT��is��typically��the��imaging
modality��of�choice��if�there��is��significant��concern��about��serious��pathology�
�or��if��the��diagnosis��is��unclear��from
the��history,��physical��examination,��and��laboratory��testing.��In��this��instanc
e,��the��American��College��of
Radiology��recommends��CT��of��the��abdomen��and��pelvis,��preferably��with��contra
st.��One��prospective��study��of
patients��with��nontraumatic��abdominal��pain��in��an��emergency��department��setti
ng��found��that��CT��results
changed��the��leading��diagnosis��in��49%��of��patients��and��the��management��plan
��in��42%��of��patients.
12

Item��36

Family��physicians��are��faced��with��many��treatment��options��for��managing��chro
nic��low��back��pain.��Both
supervised��walking��and��physical��therapy��resulted��in��decreased��disability��a
nd��pain��scores��at��6��months��in��a
2015��randomized,��controlled��trial.��A��walking��program��is��the��best��choice��
because��it��is��much��more��affordable
than��physical��therapy.��Cochrane��reviews��found��that��neither��epidural��cortic
osteroid��injections��nor
transcutaneous��electrical��nerve��stimulation��were��better��than��placebo,��and��
spinal��manipulation��showed��no
benefit��after��6��months.��Spinal��cord��stimulators��may��help��patients��with��s
evere��pain��after��failed��back��surgery,
but��lack��moderate��or��better��evidence��to��support��their��widespread��use.

Item��37

Agranulocytosis��occurs��in��about��1��in��500��patients��who��are��taking��methima
zole.��Prior��to��starting��therapy
the��patient��should��be��advised��to��be��alert��for��a��fever��and��severe��sore�
�throat.��A��recent��survey��showed��that
61��
%��of��patients��were��unaware��of��potential��adverse��reactions.��Another��seriou
s��reaction��is��hepatotoxicity.

Item��38

Medications��are��effective��for��alcohol��use��disorder��and��should��be��offered�
�in��conjunction��with��psychosocial
interventions.��Oral��naltrexone��is��the��most��effective��medication��to��prevent
��relapse.��Acamprosate��is
moderately��effective��but��is��contraindicated��in��patients��with��renal��disease
.��A��recent��meta-analysis��indicated
that��disulfiram��is��not��effective��for��preventing��relapse.��Bupropion��is��use
d��for��smoking��cessation.

13

Item��39

Geriatric��patients��are��at��greater��risk��of��adverse��drug��reactions��compared
��to��their��younger��counterparts.
Guidelines��recommend��several��antidepressant��agents��as��good��first-
line��options,��including��venlafaxine,
bupropion,��and��mirtazapine.��Among��the��SSRIs,��citalopram,��escitalopram,��and�
�sertraline��are��all��good
choices,��but��paroxetine��is��associated��with��more��anticholinergic��effects��an
d��should��be��avoided��(SOR��C).

Item��40

Since��early��intensive��behavioral��therapy��can��improve��functional��outcomes��f
or��children��with��autism��and
autism��spectrum��disorders,��early��diagnosis��is��critical.��The��American��Acade
my��of��Pediatrics��recommends
screening��all��children��with��an��autism-
specific��screening��instrument��at��18��and��24��months��in��addition��to
surveillance��of��developmental��issues��at��all��well��child��visits.

Item��41

Women��between��the��ages��of�21��and��29��at��average��risk��for��cervical��cancer
��should��be��screened��with��cytology
every��3��years��(USPSTF��A��recommendation).��Because��of��the��high��prevalence��
of��HPV��infection��in��this��age
group��and��because��there��are��no��clear��benefits��to��HPV��testing,��testing��i
s��not��recommended��(USPSTF��grade
D).��However,��most��clinicians��will��order��reflex��testing��for��high-
risk��HPV��types��if��the��Papanicolaou��smear
shows��atypical��squamous��cells��of��uncertain��significance��(ASCUS),��based��on�
�a��recommendation��by��the
American��Society��for��Colposcopy��and��Cervical��Pathology.

Women��30-
65��years��of��age��at��average��risk��for��cervical��cancer��may��be��screened��wi
th��cytology��with��HPV
cotesting��every��5��years��or��with��cytology��alone��every��3��years��(USPSTF��gr
ade��A).��HPV��DNA��typing��and
colposcopy��are��not��screening��tests��and��are��used��for��further��evaluation��o
f��cytologic��abnormalities.��HPV
antibody��results��have��no��role��in��screening.��Women��with��a��history��of��HPV
��immunization��should��continue
to��be��screened��according��to��usual��guidelines.

14
Item��42

Third��degree��atrioventricular��(AV)��block��is��characterized��by��the��inability
��of��atrial��impulses��to��reach��the
ventricles.��Hence,��the��EKG��will��show��both��atrial��activity��(P��waves)��and�
�ventricular���escape���activity��(QRS
complexes)��with��no��correlation��between��them.��Hemodynamically��unstable��patie
nts��with��third��degree��AV
block��should��be��emergently��treated��with��atropine��and��temporary��pacing.��Wi
th��Mobitz��type��I��AV��block��there
is��progressive��PR��interval��prolongation��preceding��a��nonconducted��P��wave.��
With��Mobitz��type��II��AV��block
the��PR��interval��remains��the��same��but��there��is��a��sudden��dropped��P��wave.
��Second��and��third��degree��AV��blocks
are��seen��in��patients��with��advanced��heart��disease.��The��EKG��in��atrial��fib
rillation��lacks��distinct��P��waves.
Additionally,��fibrillary��waves��and��irregularly��irregular��intervals��between��
QRS��complexes��are��characteristic.
Item��43

Neurologic��complications��related��to��Paget�s��disease��of��the��bone��can��resul
t��from��bony��compression��of��the
cranial��nerves,��spinal��cord,��and/or��nerve��roots,��resulting��in��pain��or��lo
ss��of��function,��or��from��the��creation
of��a��vascular��steal��of��blood��supplying��CNS��structures,��which��causes��para
lysis��that��is��often��reversible.��In
rare��cases��it��is��caused��by��basilar��invagination��of��the��C2��vertebra��comp
ressing��the��brain��stem��and��leading
to��hydrocephalus.��Hearing��loss,��the��most��common��neurologic��complication��of
�Paget�s��disease,��was��initially
believed��to��be��caused��by��compression��of��the��vestibulocochlear��nerve��but��
is��now��thought��to��be��the��result��of
cochlear��damage.

Item��44

This��patient�s��presentation��and��mechanism��of��injury��suggest��radial��head��s
ubluxation,��or���nursemaid�s
elbow.���This��is��a��clinical��diagnosis��that��usually��does��not��require��imagi
ng��prior��to��attempted��reduction,
either��by��hyperpronation��or��the��supination��and��flexion��method.��There��is��
no��evidence��of�a��fracture��requiring
immobilization��of��the��joint��or��the��arm,��but��the��injury��does��require��tre
atment.
15
Item��45

The��most��common��cardiac��abnormality��associated��with��Lyme��disease��is��fluct
uating��degrees��of
atrioventricular��(AV)��block.��This��may��be��first��degree,��second��degree,��or�
�complete��heart��block.��The
condition��is��usually��temporary,��lasting��from��3��to��42��days,��with��more��ad
vanced��block��lasting��longer��than
first��degree��AV��block.��The��degree��of��AV��block��can��fluctuate��within��minu
tes.��The��highest��risk��of��AV��block
is��in��patients��with��a��PR��interval��>300��msec.��Other��cardiac��abnormalities
��such��as��myocarditis,��left
ventricular��failure,��and��pericarditis��are��less��common.

Item��46

This��patient�s��test��results��suggest��a��moderate��anemia��that��is��associated�
�with��a��mild��reduction��of��platelets.
The��mean��corpuscular��volume��is��solidly��in��the��normal��range,��so��this��is�
�a��normocytic��anemia��and��is��unlikely
to��be��due��to��iron��or��folate��deficiency,��which��usually��result��in��microcy
tic��and��macrocytic��anemias,
respectively.��A��normal��ferritin��level��also��suggests��a��diagnosis��other��tha
n��iron��deficiency.

Acute��blood��loss��and��hemolysis��are��two��common��causes��of��normocytic��anemi
a.��However,��with��both��of
these��conditions��a��high��reticulocyte��count��would��be��expected.��The��fact��t
hat��the��reticulocyte��count��is��in��the
normal��range��despite��significant��anemia��suggests��that��this��patient��has��de
creased��bone��marrow��production
of��red��blood��cells,��which��is��at��least��contributing��to��his��anemia.��Bone�
�marrow��response��to��anemia��is��often
evaluated��by��using��the��reticulocyte��index,��which��is��calculated��by��multipl
ying��the��measured��reticulocyte
percentage��(1.5��in��this��patient)��by��the��ratio��of��the��patient�s��hematocri
t��to��a��normal��hemotocrit��based��on��the
person�s��age��and��sex��(26.9��divided��by��45��in��this��case).��If��the��patient
��has��a��normal��bone��marrow,��the
reticulocyte��index��should��be��2%-
3%��or��higher.��In��this��case��the��result��is��0.9%,��suggesting��a��profound
suppression��of��bone��marrow.

Item��47

This��patient��is��more��likely��to��have��bacterial��vaginosis��than��other��cause
s��of��vaginitis,��based��on��the��presence
of��a��foul-
smelling��discharge��in��the��absence��of��pruritus��or��dyspareunia.��The��odor��o
f��bacterial��vaginitis��is
more��likely��to��appear��in��the��presence��of��semen��because��of��the��increase�
�in��vaginal��alkalinity.��Bacterial
vaginosis��is��associated��with��a��vaginal��pH��>4.5.��Vulvovaginal��candidiasis��
is��associated��with��vulvar
erythema,��excoriation��of��the��vulva,��and��a��normal��vaginal��pH��(4.0-
4.5).��Vaginal��atrophy��is��seen��in��women
with��estrogen��deficiency,��and��atrophic��vaginitis��is��unlikely��in��this��pati
ent��due��to��her��age��and��lack��of�pruritus
and��dyspareunia.

16

Item��48

Pregnant��women��should��receive��a��dose��of�Tdap,��optimally��between��27��and��3
6��weeks��of�gestation,��regardless
of��when��they��last��received��it.��MMR��vaccine��and��varicella��vaccine��are��bo
th��live��attenuated��vaccines��and
should��not��be��given��to��women��known��to��be��pregnant.��Influenza��vaccine��is
��recommended��for��all��women��who
are��or��will��be��pregnant��during��influenza��season,��but��pregnant��women��shou
ld��receive��the��inactivated
influenza��vaccine.
Item��49

According��to��a��Cochrane��review,��routine��preoperative��testing��prior��to��cat
aract��surgery��does��not��decrease
intraoperative��or��postoperative��complications��(SOR��A).��The��American��Heart��
Association��recommends
against��routine��preoperative��testing��in��asymptomatic��patients��undergoing��lo
w-risk��procedures,��since��the
cardiac��risk��associated��with��such��procedures��is��less��than��1��%

Item��50

This��patient��has��gout,��based��on��his��history��and��examination.��NSAIDs,��col
chicine,��or��corticosteroids��may
be��used��as��first-
line��treatment��of��gout��(SOR��B).��The��best��initial��treatment��in��this��pati
ent��would��be
prednisone,��due��to��his��advanced��chronic��kidney��disease.��Allopurinol��may��b
e��a��good��choice��to��prevent��future
episodes��if�his��uric��acid��level��is��elevated,��but��should��not��be��started��
as��an��acute��treatment.��Probenecid��would
also��not��be��an��acute��treatment��but��can��be��used��in��some��patients��for��p
revention��of��future��attacks.

17

Item��51
Severe��hyponatremia��with��symptoms��of�confusion��and��seizures��requires��raisin
g��the��serum��sodium��level��until
symptoms��improve.��Symptomatic��hyponatremia��occurs��when��sodium��levels��decrea
se��over��less��than��24
hours.��Once��symptoms��resolve,��the��cause��should��be��determined.��The��rate��o
f��sodium��correction��should��be
6-
12��mEq/L��in��the��first��24��hours��and��18��mEq/L��or��less��in��the��first��48�
�hours.��An��increase��of��4-6��mEq/L
is��usually��sufficient��to��reduce��symptoms��of��acute��hyponatremia.��Rapid��cor
rection��of��sodium��levels��can
result��in��osmotic��demyelination��(previously��called��central��pontine��myelinol
ysis).

Infusion��of��normal��saline��or��0.45%��saline��will��not��correct��the��sodium��a
s��rapidly��as��3%��saline��in��acute,
severe��hyponatremia.��Desmopressin,��1-2��pg��every��4-
6��hours,��can��be��used��concurrently��with��3%��saline.

Item��52

After��cleansing��with��saline��or��tap��water,��application��of��a��moist,��nonadh
esive��bandage��is��the��preferred��way
to��manage��a��pressure��ulcer.��A��moist��wound��environment��assists��in��healing
��and��aids��in��autolytic
debridement.��Wet-to-
dry��dressings��may��impede��healing��by��causing��pain��and��unnecessary��debridem
ent��when
a��fully��dry��dressing��is��removed.��Chlorine-based��and��povidone-
iodine��solutions��should��be��avoided��because
they��may��impede��granulation��tissue��formation.��While��it��is��important��to��d
ebride��necrotic��tissue��and��slough,
a��dry,��intact��eschar��over��the��heels��without��any��sign��of��infection��shoul
d��be��left��in��place��as��a��natural��biologic
cover.

Item��53

Anticoagulant��medications��have��been��consistently��identified��as��the��most��co
mmon��cause��of��adverse��drug
events��across��health��care��settings��in��the��United��States.��The��top��three��
categories��responsible��for��adverse��drug
events��are��anticoagulants,��opioids,��and��diabetic��agents.

18
Item��54

Shoulder��pain��is��the��third��most��common��musculoskeletal��reason��patients��co
nsult��primary��care��physicians,
and��rotator��cuff��disease��is��the��most��common��cause��of��shoulder��pain.��Pai
n��and��restricted��active��and��passive
range��of��motion,��accompanied��by��pain��and��joint��stiffness,��are��diagnostic�
�of��adhesive��capsulitis.��Range��of
motion��would��not��likely��be��affected��with��a��partial��rotator��cuff��tear,��s
ubdeltoid��bursitis,��active��myositis,��or
osteoarthritis.��Limitations��that��occur��only��with��active��motion��suggest��imp
airment��of��rotator��cuff��muscles.

Item��55

An��incidentally��discovered��adrenal��mass��is��a��common��finding��on��abdominal�
�CT��and��MRI,��occurring��in
approximately��3%-4%��of��scans.��However,��only��about��1��
%��of��these��are��malignant,��and��malignancies��rarely
occur��in��lesions��<5��cm��in��size.��Metastatic��lesions��are��rare��in��patients
��without��a��history��of��cancer.��An
incidentally��discovered��adrenal��mass��1-
4��cm��in��size��is��most��likely��to��be��a��benign��adenoma,��although
follow-up��is��often��indicated��to��ensure��stability��of��the��lesion.

Item��56

There��are��multiple��risk��factors��for��uncomplicated��cases��of�cystitis.��Sexua
l��intercourse��is��the��most��common.
Others��include��spermicide��use,��previous��urinary��tract��infection,��a��new��se
x��partner,��and��a��family��history��of
urinary��tract��infections��in��a��first��degree��female��relative.

Various��studies��have��shown��no��relationship��between��cystitis��and��water��con
sumption,��urinating��after
intercourse,��patterns��of��wiping��after��urination,��use��of��hot��tubs,��type��o
f��underwear,��or��obesity.

19

Item��57

Infants��less��than��12��months��of��age��have��higher��rates��of��pertussis��infec
tion��and��have��the��largest��proportion
of�pertussis-
related��deaths.��The��majority��of�pertussis��cases,��admissions,��and��deaths��oc
cur��in��children��under
2��months��of��age��before��they��receive��their��first��vaccines.��The��Advisory��
Committee��on��Immunization
Practices��recommends��that��all��unvaccinated��family��members��get��a��dose��of�T
dap��to��help��protect��infants��from
pertussis.��Both��13-valent��and��23-
valent��pneumococcal��vaccine��are��indicated��for��someone��over��65��years��of
age,��but��there��is��not��a��demonstrated��benefit��for��the��health��of��the��inf
ant��in��this��case.��Meningococcal��and
varicella��vaccines��are��not��routinely��given��to��adults��over��age��65��and��ha
ve��not��been��shown��to��help��protect��the
infants��they��have��contact��with.

Item��58

The��U.S.��Preventive��Services��Task��Force��recommends��against��screening��for��
asymptomatic��carotid��artery
stenosis��(grade��D��recommendation),��citing��with��moderate��certainty��that��ris
ks��outweigh��benefits.��Although
carotid��artery��stenosis��is��a��risk��factor��for��stroke,��which��is��a��major��
cause��of��death��and��disability,��screening
tests��were��not��found��to��improve��patient��outcomes.

Asymptomatic��carotid��artery��stenosis��has��a��low��prevalence��(0.5%-l��
%)��and��carotid��ultrasonography��has��a
high��rate��of��false-
positives,��exposing��patients��to��harm��from��unnecessary��treatment.��Surgical��
treatments��for
carotid��artery��stenosis��have��a��30-
day��risk��of��stroke��and��mortality��of��2.2%-3.8%.��Carotid��auscultation��has
not��been��found��to��be��accurate��or��beneficial,��and��screening��has��not��been
��shown��to��help��optimize��medical
therapy.

Item��59

In��spite��of��good��evidence��that��antibiotics��are��ineffective��for��the��treat
ment��of��acute��bronchitis,��and��that��90%
of��cases��are��caused��by��viruses,��rates��of��antibiotic��prescription��for��acu
te��bronchitis��remain��in��the��60%-80%
range.��Several��strategies��have��been��shown��to��reduce��the��rate��of��antibiot
ic��prescribing��for��this��condition.
These��include��careful��use��of��nonmedical��terminology��such��as��referring��to�
�the��problem��as��a���chest��cold,�
providing���pocket���prescriptions��with��advice��to��fill��the��prescription��only
��if��the��patient��does��not��improve
in��a��defined��period��of��time,��and��educating��patients��about��the��natural��h
istory��of��bronchitis,��informing��them
that��symptoms��may��persist��for��3��weeks.��Specialists��are��not��less��likely��
than��primary��care��physicians��to
prescribe��antibiotics.��Sinus��films��would��not��provide��evidence��to��confirm��
that��the��infection��is��viral.

2
Item��60

NSAIDs��should��be��used��as��first-
line��treatment��for��primary��dysmenorrhea��(SOR��A).��A��Cochrane��review��that
included��73��randomized,��controlled��trials��demonstrated��strong��evidence��to��
support��NSAIDs��as��the��first-line
treatment��for��primary��dysmenorrhea.��Since��no��NSAID��has��been��proven��more��
effective��than��others,��the
choice��of��NSAID��should��be��based��on��effectiveness��and��tolerability��for��ea
ch��patient.��The��medication��should
be��taken��1-
2��days��before��the��expected��onset��of��the��menstrual��period��and��continued��
on��a��fixed��schedule��for
2-
3��days.��Oral��contraceptives��may��be��effective��for��relieving��symptoms��of��p
rimary��dysmenorrhea��but��the
evidence��is��limited.��Hydrocodone,��acetaminophen,��and��medroxyprogesterone��ace
tate��are��not��appropriate
choices.

Ref:

Osayande��AS,��Mehulic��S:��Diagnosis��and��initial��management��of��dysmenorrhea.�
�Am��Fam��Physician��2014;89(5):341-346.

Item��61

Less��than��1��
%��of��patients��started��on��an��ACE��inhibitor��develop��angioedema,��but��some��
studies��have��reported
that��up��to��10%��of��these��patients��require��intubation.��This��type��of��angio
edema��is��due��to��increased��bradykinin
rather��than��histamine,��and��antihistamines,��anticholinergics,��corticosteroids,
��and��epinephrine��would��not��be
effective.��Icatibant��is��a��bradykinin��receptor��type��2��blocker��and��is��reco
mmended��in��patients��with��laryngeal
angioedema��compromising��airway��function��(level��2��evidence).��Angiotensin��rec
eptor��blockers,��although
probably��not��harmful,��would��not��be��helpful.

Item��62

The��euthyroid��sick��syndrome��refers��to��alterations��in��thyroid��function��tes
ts��seen��frequently��in��hospitalized
patients,��and��abnormal��thyroid��function��tests��may��be��seen��early��in��sepsi
s.��These��changes��are��statistically
much��more��likely��to��be��secondary��to��the��euthyroid��sick��syndrome��than��to
��unrecognized��pituitary��or
hypothalamic��disease��(SOR��C).��Graves��disease��generally��is��a��hyperthyroid��
condition��associated��with��low
TSH��and��elevated��free��T4.��Subclinical��hypothyroidism��is��diagnosed��by��high
��TSH��and��normal��free��T4��levels.
Subacute��thyroiditis��most��often��is��a��hyperthyroid��condition.
21

Item��63

Endoscopic��screening��results��in��the��detection��of�Barrett�s��esophagus��in��6%
-12%��of�patients��with��prolonged
gastroesophageal��reflux��disease��symptoms.��Barrett�s��esophagus,��in��which��spe
cialized��intestinal��columnar
epithelium��replaces��the��normal��esophageal��lining��in��response��to��chronic��i
nflammation,��is��a��precursor��of
esophageal��adenocarcinoma.��The��annual��cancer��risk��for��patients��with��nondys
plastic��Barrett�s��esophagus��is
0.12%-
0.4%,��with��a��significant��increase��in��risk��if��dysplasia��is��present.��Surve
illance��with��endoscopy��every
3��years��is��recommended��for��patients��with��Barrett�s��esophagus��without��dysp
lasia.��Patients��with
adenocarcinoma��of�the��esophagus��found��during��surveillance��endoscopy��are��mor
e��likely��to��have��early-stage,
curable��cancer��than��those��whose��cancer��is��found��during��a��diagnostic��endo
scopy��for��evaluation��of�symptoms.

Item��64

If��TSH��is��suppressed��in��this��patient��it��indicates��that��the��nodule��is��p
roducing��thyroid��hormone��and��further
evaluation��with��a��radionuclide��scan��is��indicated.��If��the��TSH��is��normal��
or��elevated��the��next��step��is��to
determine��whether��the��nodule��needs��to��be��biopsied.��Thyroid��ultrasonography
��can��determine��the��size��and
characteristics��of��the��nodule��to��help��determine��whether��to��refer��the��pat
ient��for��a��fine-needle��aspiration
biopsy.��With��a��large,��firm��lesion��that��is��highly��suspicious��for��malignan
cy,��it��may��be��appropriate��to��refer
directly��for��a��fine-
needle��aspiration��biopsy.��However,��for��this��patient��the��lesion��did��not��a
ppear��suspicious.
If��the��patient��were��hyperthyroid��it��might��be��appropriate��to��check��antith
yroid��antibodies��to��look��for��Graves
disease.

Item��65

Keloids��are��overgrowths��of��scar��tissue��seen��more��commonly��in��individuals�
�with��dark��skin.��The��best��initial
treatment��is��intralesional��corticosteroid��injections.��If��this��does��not��pro
duce��acceptable��results,��other
treatment��modalities��include��surgery,��laser��therapy,��and��bleomycin��injectio
n.
22

Item��66

The��dissociative��reactions��(flashbacks)��in��this��patient��are��consistent��wit
h��the��diagnosis��of��posttraumatic
stress��disorder��(PTSD).��The��first-
line��medications��for��this��disorder��are��SSRIs��and��SNRIs.��Paroxetine��and
sertraline��have��FDA��approval��for��PTSD.��Other��antidepressants��such��as��mirt
azapine��would��be��second-line
therapy.��The��effectiveness��of��central��cc2-
agonists��such��as��clonidine��are��unknown,��and��even��though
benzodiazepines��might��help��with��hyperarousal��symptoms,��they��can��worsen��oth
er��symptoms.��Atypical
antipsychotics��such��as��risperidone��are��not��recommended.

Item��67

Crohn�s��disease��typically��spares��the��rectum.��Ulcerative��colitis��usually��ha
s��rectal��involvement,��progresses
proximally,��and��rarely��has��perianal��or��systemic��manifestations��(SOR��A).��B
oth��conditions��may��cause��bloody
diarrhea��as��well��as��inflammatory��arthropathies,��eye��inflammation��such��as��
uveitis,��and��skin��findings��such
as��erythema��nodosum.

Item��68

Many��drugs��can��cause��lung��disease.��Amiodarone��has��been��known��to��cause��b
oth��bronchiolitis��obliterans
organizing��pneumonia��(BOOP)��and��interstitial��pneumonitis.��BOOP,��also��known�
�as��cryptogenic��organizing
pneumonia,��is��characterized��by��interstitial��inflammation��superimposed��on��th
e��dominant��background��of
alveolar��and��ductal��fibrosis.��This��is��a��very��distinctive��pattern��of�lung�
�response��to��exposure��to��several��drugs,
including��amiodarone,��bleomycin,��gold,��penicillamine,��sulfasalazine,��radiatio
n,��interferons,��methotrexate,
mitomycin��C,��cyclophosphamide,��and��cocaine.

Interstitial��pneumonitis��is��the��most��common��manifestation��of��drug-
induced��lung��disease.��Drugs��that��can
cause��this��include��amiodarone,��azathioprine,��bleomycin,��chlorambucil,��methot
rexate,��phenytoin,��statins,
and��sulfasalazine.
23

Item��69

This��patient��demonstrates��an��antalgic��gait��without��a��clear��etiology.��The�
�evaluation��of��a��limping��child��begins
with��a��thorough��history,��observation��of��the��child�s��gait,��and��a��physical
��examination.��If��the��history��is��not
contributory��and��the��physical��examination��demonstrates��no��focal��source��of�
pain,��radiographs��of�both��lower
extremities��should��be��the��first��step��in��the��workup��(SOR��C).��If��there��i
s��a��focal��source��of��pain,��radiographs
of��the��affected��joint��would��be��appropriate��(SOR��C).��If��the��patient��demo
nstrates��systemic��signs��of��illness
such��as��fever��or��anorexia,��a��laboratory��evaluation��(CBC,��erythrocyte��sedi
mentation��rate,��and��C-reactive
protein)��should��be��performed��in��addition��to��radiographs.��Ultrasonography��i
s��useful��subsequently��if��there
is��concern��about��joint��effusion.��If��no��source��of��the��problem��is��found,�
�additional��testing��should��include��a
bone��scan��(SOR��C)��and��MRI.

Item��70

Men��who��have��sex��with��men��but��are��in��a��monogamous��relationship��need��no
t��be��offered��preexposure��or
postexposure��HIV��prophylaxis,��unlike��men��with��multiple��or��anonymous��sexual
��partners.��Meningococcal
vaccine��is��not��indicated��unless��there��are��other��risk��factors.��Since��this
��patient��is��in��a��monogamous
relationship,��screening��for��sexually��transmitted��infections��once��a��year��is
��considered��adequate.��Screening
for��hepatitis��C��at��this��visit��is��recommended,��as��well��as��testing��for��h
epatitis��B��infection.

Item��71

Based��on��the��patient�s��age��it��is��appropriate��for��her��to��be��screened.��H
er��bone��density��is��consistent��with
osteopenia��and��she��has��no��identified��secondary��causes��of��osteopenia.��Beca
use��her��estimated��10-year��risk
of��hip��fracture��is��<3%��and��she��has��not��had��any��fractures��to��date,��pre
scription��medications��such��as
bisphosphonates��or��calcitonin��are��not��indicated.��Estrogen��does��increase��bo
ne��density��but��it��is��not��indicated
for��osteoporosis��prevention��or��treatment,��due��to��associated��cardiovascular�
�risks.��Professional��organizations
vary��on��the��daily��calcium��and��vitamin��D��intake��recommended��for��postmenop
ausal��women,��but��1200��mg
of��dietary��calcium��and��supplementation��with��800��IU��of��vitamin��D3��are��re
asonable��recommendations.��The
National��Osteoporosis��Foundation��suggests��treatment��of��osteoporosis��if��the�
�10-year��risk��of��major
osteoporotic��fracture��is��>20%.

Item��72

Meniscal��tears��are��a��common��source��of��knee��pain��in��acute��knee��injuries,
��occurring��in��approximately��10%
of��cases��presenting��with��acute��pain��after��an��injury.��Although��the��McMurr
ay��test��(passive��extension��of��the
knee��while��applying��valgus��and��varus��stresses��to��the��knee)��has��historica
lly��been��used��to��detect��meniscus
injuries,��the��Thessaly��test��has��superior��positive��and��negative��predictive�
�value��for��meniscus��injuries
compared��with��the��McMurray��test.��The��Thessaly��test��is��performed��by��havin
g��the��patient��stand��on��the
affected��leg��while��it��is��flexed��20���and��internally��and��externally��rotate
��the��knee��three��times��while��holding
the��examiner�s��hands��for��support.��Locking,��catching,��or��joint-
line��pain��constitutes��a��positive��test.��The
pivot-
shift,��Lachman,��and��anterior��drawer��tests��are��used��to��detect��injuries��to
��the��anterior��cruciate��ligament,
not��meniscal��injuries.��The��valgus��stress��test��detects��injuries��to��the��me
dial��collateral��ligament.

Item��73

This��patient��has��low��energy,��menstrual��irregularities,��and��a��history��of��
stress��fractures,��which��is��consistent
with��the��female��athlete��triad.��She��is��at��risk��for��altered��bone��density�
�and��needs��a��bone��mineral��density��test.
This��condition��is��also��associated��with��disordered��eating��and��low��body��ma
ss.��Echocardiography��may��be
indicated��if��there��is��a��personal��or��family��history��of��cardiac��problems.�
�Radiographs��of��the��feet��are��not
sensitive��for��osteoporosis��evaluation.��A��stress��test��is��not��indicated.

Item��74

Over��85%��of��cases��of��infectious��endocarditis��are��caused��by��gram-
positive��cocci.��In��patients��suspected��of
having��acute��infectious��endocarditis,��empiric��antibiotic��treatment��should��b
e��started��immediately��after
obtaining��initial��blood��cultures��and��should��include��coverage��against��gram-
positive��cocci��with��vancomycin.
For��patients��with��prosthetic��heart��valves,��initial��coverage��should��include
��vancomycin��plus��rifampin.
Clindamycin,��levofloxacin,��and��trimethoprim/sulfamethoxazole��have��no��role��in
��the��initial��treatment��of
infectious��endocarditis.

25

Item��75

Vaccination��rates��for��younger��children��(4-
6��years��of�age)��generally��surpass��90%,��but��rates��are��much��lower
in��older��children.��Only��34%��of��boys��receive��HPV��vaccine��at��age��11��or��
12,��and��only��40%��receive��Tdap.
Family��physicians��need��to��be��familiar��with��routine��immunizations��in��this�
�age��group��so��they��can��be
recommended��at��the��appropriate��time.

This��patient��was��up��to��date��with��immunizations��at��age��6��years,��so��the�
�only��catch-up��vaccines��needed��are
those��in��the��routine��11-��to��12-year-
old��set,��which��includes��influenza,��meningococcal,��HPV,��and��Tdap
vaccines.��DTap��and��varicella��vaccine��are��not��routinely��recommended��after��
the��age��of��6��years.
Item��76

Early��studies��of��ischemic��heart��disease��included��mostly��male��subjects.��Mo
re��recently��there��has��been��a
determined��effort��to��understand��the��special��considerations��associated��with�
�this��problem��and��its��management
in��women.��In��2014��the��American��Heart��Association��published��a��consensus��s
tatement��summarizing��the
research��on��how��to��best��evaluate��women��with��suspected��ischemic��heart��dis
ease.��Its��recommendations
focused��on��the��level��of��pretest��risk��for��ischemic��heart��disease��(low,��i
ntermediate,��and��high),��a��normal��or
abnormal��resting��EKG,��the��ability��of��the��subject��to��exercise,��and��potent
ial��risks��of��radiation��exposure.��The
patient��in��this��scenario��would��be��considered��low��to��intermediate��risk��fo
r��ischemia��due��to��her��age��and��risk
factors,��along��with��a��history��of��atypical��chest��pain.��Since��her��resting�
�EKG��is��normal��and��she��is��physically
fit,��she��should��undergo��an��exercise��treadmill��test��without��imaging.
26

Item��77

Gastroesophageal��reflux��is��very��common��in��infants.��Reflux��in��infants��peak
s��at��4��months��of��age,��and
two-
thirds��of��infants��regurgitate��at��least��once��a��day��at��this��age.��The��inc
idence��declines��dramatically��in��the
next��few��months,��and��by��1��year��of��age��less��than��5%��of��infants��regurgi
tate��on��a��daily��basis.��If��the��infant��is
healthy��and��growing��normally,��reassurance��is��appropriate��(SOR��C).��Conserva
tive��measures��are
recommended��if��the��reflux��causes��distress��(SOR��C).��Such��measures��include�
�placing��the��infant��on��her��side
or��prone��while��awake��to��reduce��reflux.��However,��infants��should��not��be��p
laced��prone��while��sleeping,��to
prevent��SIDS.��Appropriate��conservative��measures��also��include��smaller,��more�
�frequent��feedings,��the��addition
of��thickening��agents��such��as��rice��cereal��to��formula,��and��changing��to��am
ino��acid��formulas��if��infants��are
allergic��to��cow�s��milk��protein.

If��conservative��measures��and��time��do��not��alleviate��the��symptoms,��a��4-
week��trial��of��H2-blockers��or��proton
pump��inhibitors��can��be��tried.��The��American��Academy��of��Pediatrics��and��the
��Society��of��Hospital��Medicine
(Pediatric) recommend not routinely treating reflux with medication.
Imaging, such as
esophagogastroduodenoscopy,��is��not��routinely��used��in��the��initial��workup��of
��reflux��in��infants.��It��is��reserved
for��recalcitrant��cases,��atypical��symptoms,��or��complications��(SOR��C).

Item��78

Henoch-
Schonlein��purpura��(HSP)��presents��most��often��in��children��but��not��infrequen
tly��in��adults.��The
purpuric��rash��is��classically��seen��on��the��waist��and��extends��to��the��legs,
��sparing��the��proximal��trunk��and��arms.
Orchitis��with��testicular��swelling��occurs��in��35��
%��of��men��with��HSP��and��is��often��complicated��by��abdominal
pain,��arthritis,��and��renal��insufficiency.

Kawasaki��disease��is��a��pediatric��disease��presenting��with��fever,��conjunctivi
tis,��and��lesions��of��the��lips.
Polyarteritis��nodosa��often��presents��with��fever��and��multisystem��symptoms��an
d��findings.��There��are��most��often
abnormalities��on��the��CBC��and��chemistry��profile.��Rocky��Mountain��spotted��fe
ver��is��associated��with��a
petechial��rash��that��involves��the��proximal��trunk��and��extremities,��including
��the��palms��and��soles.
Thrombocytopenic��purpura,��by��definition,��is��associated��with��a��low��platelet
��count.
27

Item��79

Opioids,��given��either��orally��or��intravenously,��are��the��treatment��of�choice
��for��dyspnea��and��have��been��studied
thoroughly��in��patients��with��COPD��and��patients��with��cancer.��They��have��bee
n��found��to��be��effective��in
alleviating��dyspnea��and,��when��used��carefully,��do��not��have��serious��side��e
ffects��such��as��respiratory
depression.��When��the��patient��is��experiencing��anxiety,��which��regularly��occu
rs��in��association��with
breathlessness,��benzodiazepines��can��be��added,��although��there��is��no��evidenc
e��that��they��improve��the��dyspnea.
Patients��are��regularly��given��supplemental��oxygen��for��dyspnea,��but��systemat
ic��reviews��have��found��no��benefit
for��patients��with��cancer��or��heart��failure��who��do��not��have��hypoxemia.��Ho
wever,��oxygen��may��provide��some
relief��for��patients��with��COPD��who��do��not��have��hypoxemia.��Prednisone��and�
�albuterol��are��not��indicated��for
this��patient.

Item��80

Core��features��of��the��behavioral��variant��frontotemporal��dementia��(FTD)��incl
ude��an��insidious��onset��and
gradual��progression,��an��early��decline��in��social��and��interpersonal��conduct,
��early��impairment��in��regulation
of�personal��conduct,��early��emotional��blunting,��and��early��loss��of��insight.�
�Common��initial��symptoms��include
apathy,��lack��of��initiation,��diminished��interest,��and��inactivity.��Common��fe
atures��also��include��disinhibition
and��impulsivity.��Examples��include��socially��inappropriate��remarks,��including�
�sexual��comments.

These��types��of��symptoms��are��less��common��in��early��phases��of��other��types�
�of��dementia.��FTD��is��frequently
misdiagnosed��as��a��primary��psychiatric��disorder��such��as��depression.��Alzheim
er�s��disease��presents��with
memory��and��visuospatial��loss.��Lewy��body��dementia��tends��to��cause��memory��l
oss,��fluctuating��cognition,
visual��hallucinations,��and��spontaneous��parkinsonian��motor��features.��Vascular
��dementia�patients��usually��have
a��history��of�cerebrovascular��events.��Mixed��dementias��generally��are��a��combi
nation��of�Alzheimer�s��and��other
types��of��dementias.
Item��81

The��mainstay��of��treatment��for��osteoarthritis��of��the��knee��is��active��rehab
ilitation��and��exercise��(SOR��A).
Active��rehabilitation,��such��as��stretching��and��strengthening,��is��more��effec
tive��than��passive��rehabilitation,
such��as��taping,��heat,��electrostimulation,��or��therapeutic��ultrasound��(SOR��B
).��Lateral��wedge��insoles��and
glucosamine��and��chondroitin��supplements��are��unlikely��to��significantly��impro
ve��pain��in��patients��with��knee
arthritis��(SOR��B).

28

Item��82

This��child��meets��the��criteria��for��acute��bacterial��sinusitis��(ABS)��and��sh
ould��be��treated��with��antibiotics.��He
exemplifies��the��concept��of���double��sickening,���in��which��a��child��initially
��has��typical��symptoms��of��a��viral
upper��respiratory��infection��and��improves��initially��only��to��worsen��later,��
with��daytime��cough,��persistent��nasal
discharge,��and/or��new��fever.��Other��criteria��for��ABS��include��persistence��o
f��URI��symptoms��without
improvement��after��7-
10��days��and���severe��onset���ABS��with��a��high��fever��and��purulent��nasal��di
scharge��for
at��least��3��days.��Evidence��shows��that��treatment��with��antibiotics��in��these
��situations��improves��outcomes��(SOR
B).��The��first-
line��antibiotic��is��amoxicillin��with��or��without��clavulanate.��The��length��of
��treatment��can��range
from��10��to��28��days.��Depending��on��risks,��patients��may��be��treated��with��e
ither��high-dose��amoxicillin��or
amoxicillin/clavulanate,��with��an��amoxicillin��dosage��of��90��mg/kg/day.��Many��
of��the��bacteria��causing��ABS
have��been��shown��to��be��resistant��to��azithromycin��and��trimethoprim/sulfameth
oxazole��and��these��antibiotics
should��be��avoided.��For��patients��allergic��to��penicillins,��cephalosporins��sh
ould��be��used.

The��diagnosis��of�ABS��is��based��on��the��history��(SOR��C).��The��physical��exam
ination��is��not��particularly��helpful
and��findings��such��as��sinus��tenderness,��mucosal��swelling,��and��transillumina
tion��of��the��sinuses��do��not��help
differentiate��ABS��from��a��viral��URI.��Laboratory��studies��are��not��indicated�
�in��the��diagnosis.��Imaging��studies
are��likewise��not��indicated��for��the��initial��diagnosis,��as��they��are��often�
�abnormal��in��both��viral��URIs��and��ABS.
If��complications��such��as��orbital��cellulitis��or��neurologic��compromise��are��
a��concern,��then��CT��may��be
indicated.

There��is��no��good��evidence��to��support��adjuvant��care��for��ABS.��Saline��nasa
l��irrigation,��decongestants,��or
intranasal��corticosteroids��may��be��helpful��but��cannot��replace��antibiotic��th
erapy��in��children��who��meet��the
criteria��for��ABS.
Item��83

The��2014��evidence-
based��guideline��from��the��JNC��8��panel��recommends��that��in��the��general��Afr
ican-American
population,��including��those��with��diabetes��mellitus,��initial��antihypertensive
��treatment��should��include��a
thiazide-type��diuretic��or��calcium��channel��blocker��(for��general��African-
American��population:��SOR��B;��for
African-American��patients��with��diabetes:��SOR��C).

29

Item��84

Asthmatic��symptoms��during��exercise��are��common.��These��can��occur��as��exacerb
ations��of�underlying��airway
inflammation��or��as��bronchospasm��in��otherwise��normal��airways.��This��patient�
�seems��to��have��bronchospasm
that��would��be��best��managed��by��albuterol��prior��to��exercise.��If��she��finds
��that��she��needs��her��inhaler��frequently,
the��addition��of��an��anti-
inflammatory��agent��such��as��inhaled��corticosteroids��or��oral��montelukast��wou
ld��be
reasonable.

Item��85

Ankylosing��spondylitis��is��an��inflammatory��condition��that��affects��the��axial
��skeleton��primarily,��but��other
joints��may��be��involved.��The��pain��begins��insidiously��in��the��lower��back��a
nd��gluteal��region.��The��symptoms
improve��with��exercise��and��activity��but��they��worsen��at��night.��The��stiffne
ss��is��most��prominent��in��the��morning
and��may��last��up��to��a��few��hours.

Item��86

Radiation��exposure��in��humans��is��quantified��by��the��sievert��(Sv),��which��eq
uals��1��joule��of�radiation��energy/kg
of��human��tissue.��Most��clinical��diagnostic��test��exposures��measure��in��the��
millisievert��(mSv)��range.��Natural
background��exposure��in��the��United��States��averages��3��mSv/year.��Although��ex
act��exposures��will��vary
according��to��patient��size,��type��of��equipment��used,��and��operator��expertise
,��typical��radiation��doses��for
common��radiographic��studies��include��the��following:

Posteroanterior��chest�0.02��mSv
Skull�0.1��mSv
Lumbar��spine�1.5��mSv
CT��head�2��mSv
CT��abdomen�8��mSv

30

This��list��shows��that��CT��of��the��abdomen��provides��a��radiation��dose��400��t
imes��that��of��the��typical
posteroanterior��chest��radiograph.��A��small��increased��relative��risk��of��cance
r��mortality��was��demonstrated��in
Japanese��survivors��of�atomic��bombs��receiving��doses��in��the��range��of�5-
20��mSv,��suggesting��that��consideration
of��the��risk/benefit��ratio��of��some��radiographic��studies��is��warranted.

Item��87

Patients��with��recurrent��nephrolithiasis��should��first��try��to��increase��fluid
��intake��to��achieve��a��daily��urine��output
>2��L.��Increasing��fluid��intake��decreases��the��recurrence��of��stones��by��at��
least��50%.��Reducing��soft��drink��intake
may��also��help��but��this��seems��to��be��limited��to��those��who��drink��colas,��
which��are��acidified��by��phosphoric��acid.
Soft��drinks��acidified��with��citric��acid,��such��as��fruit-
flavored��drinks,��do��not��appear��to��have��the��same��effect.
There��is��little��evidence��that��dietary��changes��help��significantly.��If��a��p
atient�s��fluid��increase��is��not��sufficient,
treatment��with��a��thiazide��diuretic,��citrate,��or��allopurinol��is��recommended
��(SOR��C).

Item��88

The��2013��American��College��of��Cardiology/American��Heart��Association��guidelin
e��for��reducing
atherosclerotic��cardiovascular��disease��(ASCVD)��risk��in��adults��recommended��s
everal��significant��changes
in��the��management��of��hyperlipidemia.��This��guideline��recommends��looking��at�
�overall��risk��as��estimated��by
a��tool,��the��ASCVD��Risk��Estimator,��which��considers��not��only��lipid��paramet
ers��but��also��age,��sex,��ethnicity,
systolic��blood��pressure,��and��the��presence��or��absence��of��diabetes��mellitus
,��treated��hypertension,��and
smoking.��Four��major��risk��groups��were��identified��for��treatment,��one��of��wh
ich��was��adults��>40��years��of��age
with��an��estimated��10-year��risk��of��ASCVD��>7.5%.

Therapy��is��graded��by��intensity��(low-,��moderate-,��or��high-
intensity��statin��therapy),��and��therapeutic��targets
for��LDL-
cholesterol��were��abandoned��since��there��is��no��demonstrable��benefit��from��ac
hieving��a��certain��level
of�LDL-cholesterol��in��treated��patients.��This��patient��has��an��estimated��10-
year��ASCVD��risk��well��above��7.5��%
and��is��a��candidate��for��moderate-��or��high-
intensity��statin��therapy��under��the��guideline.��Niacin��and��fibrate
therapy��do��not��have��a��demonstrable��impact��on��cardiovascular��outcomes��and�
�are��not��recommended��in��the
guideline.��PCSK9��inhibitors��are��also��not��yet��recommended��in��any��guideline
.

31

Item��89

Major��risk��factors��for��acute��mountain��sickness��include��a��history��of�previ
ous��mountain��sickness,��fast��ascent,
and��lack��of��acclimatization.��Slow��ascent��with��frequent��stops��at��various��
levels��is��the��safest��way��to��prevent
altitude��sickness.��Females��are��at��increased��risk��and��good��physical��fitnes
s��is��not��protective.��Persons��<46
years��of��age��are��at��increased��risk.

Item��90

Thiazide��diuretics��such��as��chlorthalidone��are��considered��a��first-
line��therapy��for��hypertension.��P-Blockers,
aldosterone��antagonists,��and��other��antihypertensive��medications��may��be��used
��as��add-on��therapy��to��reach
blood��pressure��goals.

Item��91

Prompt��recognition��and��treatment��of�sepsis��increases��the��chances��of�surviva
l.��Aggressive��fluid��resuscitation
is��the��initial��treatment��for��hypotension��in��patients��in��septic��shock.��An
tibiotic��therapy��should��be��administered
within��1��hour��of��suspecting��sepsis.��If��fluid��resuscitation��is��not��succes
sful��in��restoring��blood��pressure,
norepinephrine��is��the��currently��recommended��first-
line��vasopressor.��The��use��of��hydroxyethyl��starch��is��not
recommended��because��the��mortality��rate��is��higher��in��sepsis��patients.��Prev
iously,��dopamine��was
recommended,��but��low-
dose��dopamine��for��renal��perfusion��has��now��been��shown��to��be��ineffective.�
�For
patients��who��are��vasopressor��dependent,��low-
dose��corticosteroids��can��be��considered.

32

Item��92

In��spite��of��an��estimated��50%-
80%��reduction��in��cases��of��invasive��pneumococcal��disease��(IPD)��as��a��resul
t
of��vaccination��with��the��23-
valent��pneumococcal��polysaccharide��vaccine��(PPSV23),��IPD��remains��a
significant��problem��in��the��United��States,��with��approximately��40,000��cases�
�in��2010��resulting��in��about��4000
deaths.��The��introduction��of��the��13-
valent��pneumococcal��conjugate��vaccine��(PCV13)��has��been��shown��to
further��reduce��vaccine-
type��cases��of��IPD��by��as��much��as��75%,��leading��to��the��2014��recommendatio
n��from
the��Advisory��Committee��on��Immunization��Practices��to��administer��PCV13��to��a
dults��age��65��and��older.��In
immunocompetent��adults��age��65��and��older��who��are��pneumococcal��vaccine-
naive,��administering��PPSV23
1��year��or��more��after��PCV13��results��in��a��better��immune��response��than��gi
ving��PPSV23��first,��and��reduces
local��reactions��to��the��vaccines��compared��to��simultaneous��administration��or
��administering��PPSV23��first.��For
some��immunocompromised��patients��the��recommendation��is��to��wait��a��minimum��o
f��8��weeks��after��giving
PCV13��before��administering��PPSV23.

Item��93

Average-
risk��adults��should��be��screened��for��colorectal��cancer��starting��at��age��50.
��People��who��are��at��higher
risk��for��developing��colon��cancer��should��be��screened��at��either��age��40��or
��10��years��earlier��than��the��age��at
which��the��youngest��affected��family��member��was��diagnosed,��whichever��is��ear
liest.��Risk��factors��for��colon
cancer��include��age,��ethnicity,��and��family��history.��There��is��good��evidence
��that��screening��identifies
premalignant��lesions,��which��allows��for��early��treatment��and��reduced��mortali
ty.
Item��94

In��the��past,��rubella��was��a��common��cause��for��reduced��hearing.��With��the��
advent��of��vaccination,��genetic
inheritance��has��become��the��most��frequent��cause��for��deafness.��Aminoglycosid
es��are��rarely��a��reason��for
hearing��loss.��The��well-
known��association��of��aminoglycosides��with��hearing��loss��has��reduced��the��ex
posure
risk��from��these��drugs.��Head��trauma��and��prematurity��remain��important��cause
s��for��deafness��but��are��still��very
small��risk��factors��compared��to��simple��inheritance.��Prematurity��risk��has��d
iminished��with��improvements��in
the��care��of��premature��infants.

33

Item��95

Treatment��of��genital��warts��should��be��based��on��patient��preference��and��cos
t��(SOR��C).��There��is��no��treatment
that��is��more��effective��than��others,��and��the��recurrence��rate��for��any��tre
atment��is��relatively��high.��Cryotherapy
is��not��more��effective��than��other��treatments.��Treatment��in��pregnancy��has��
not��been��found��to��decrease��the��risk
of��transmission.

Item��96

Serotonin��syndrome��is��a�potentially��life-
threatening��condition��caused��by��excessive��serotonergic��activity,��and
certain��medications��are��more��likely��to��precipitate��it.��Early��recognition��
of��symptoms��is��important,��as��most
cases��can��be��managed��on��an��outpatient��basis��with��discontinuation��of��the�
�precipitating��medication/agent��and
supportive��care.��The��Hunter��Serotonin��Toxicity��Criteria��can��be��used��to��d
iagnose��serotonin��syndrome��(SOR
C).

The��differential��diagnosis��includes��anticholinergic��syndrome,��malignant��hype
rthermia,��and��neuroleptic
malignant��syndrome.

Anticholinergic��syndrome��is��associated��with��tachycardia,��tachypnea,��and
hyperthermia.��Malignant��hyperthermia��and��neuroleptic��malignant��syndrome��are�
�associated��with
hypertension,��tachycardia,��tachypnea,��and��hyperthermia.��These��conditions��are
��precipitated��by��other��classes
of��medications.��Central��cord��syndrome��is��a��spinal��cord��disease��caused��by
��spinal��trauma,��syringomyelia,
and��intrinsic��cord��tumors,��and��presents��with��arm��weakness��greater��than��l
eg��weakness.
Item��97

Only��subacute��thyroiditis��is��associated��with��a��painful��and��tender��thyroid
.��It��often��follows��an��upper
respiratory��viral��illness��that��triggers��inflammatory��destruction��of��thyroid
��tissue.��Thyroid��hormone��is
increased��and��TSH��is��suppressed.�Treatment��consists��of�corticosteroids��or��N
SAIDs.��The��condition��is��usually
self-limited.

34

Item��98

This��patient��has��a��high��risk��of��pulmonary��embolism��based��on��her��present
ation��and��the��elevated��D-dimer
assay.��CT��angiography��(CTA)��of�the��chest��would��be��the��next��step��in��the�
�evaluation��of��this��patient��(SOR��A).
The��other��tests��may��be��helpful��but��should��not��delay��chest��CTA.��If��ches
t��CTA��is��negative��a��venous��duplex
study��would��be��helpful��in��ruling��out��a��DVT.

Item��99

Retinal��detachments��can��be��caused��by��a��break��in��the��retina,��exudate��or�
�leakage��from��beneath��the��retina,��or
traction��on��the��retina.��Retinal��detachments��are��often��preceded��by��a��post
erior��vitreous��detachment,��which
can��lead��to��a��break��in��the��retina,��and��patients��may��experience��an��incr
ease��in��floaters��but��not��light��flashes
or��loss��of��vision.��With��a��retinal��detachment,��patients��typically��experien
ce��floaters��followed��within��1��week
by��flashes��of��light��as��the��retina��tears.��If��the��retinal��tear��becomes��l
arge��enough��vision��is��impaired.��Any
patient��with��the��typical��history��and��loss��of��visual��acuity��or��peripheral
��field��should��be��urgently��referred��to
an��ophthalmologist,��as��urgent��surgery��may��be��required.

Risk��factors��for��retinal��detachment��include��age��50-
75,��ocular��trauma,��previous��cataract��surgery,��family
history,��and��a��past��history��of��retinal��detachment.��A��patient��with��a��pre
vious��retinal��detachment��has��a��25%
risk��of��developing��a��retinal��detachment��in��the��other��eye.��Myopia��is��the
��other��significant��risk��factor,
associated��with��a��tenfold��increased��risk��in��patients��with��>��3��diopters��
of��refractive��error.
Central��retinal��artery��occlusion��is��manifested��by��painless��complete��loss��
of��vision��in��the��affected��eye.��Acute
angle-
closure��glaucoma��is��associated��with��eye��pain,��photophobia,��headache,��nause
a��and��vomiting,��and
cloudy��loss��of��vision,��and��the��onset��is��not��typically��associated��with��c
oughing��or��sneezing.

35

Item��100

The��2014��Global��Initiative��for��Chronic��Obstructive��Lung��Disease��(GOLD)��gu
ideline��recommends
administration��of�prednisone,��40��mg��daily��for��5��days,��for��COPD��exacerbati
ons.��The��2011��GOLD��guideline
had��recommended��the��same��dosage��of��prednisone��for��10-
14��days.��The��guideline��does��not��recommend��a
methylprednisolone��dose��pack��or��intravenous��methylprednisolone��sodium��succin
ate.��A��recent��multicenter
study��showed��that��the��shorter��duration��of��low-
dose��prednisone��was��equivalent��to��the��longer��treatment.
Corticosteroids��are��associated��with��elevated��blood��glucose;��the��development
��of�cataracts,��diabetes��mellitus,
and��osteopenia;��and��thromboembolic��complications.

Item��101

Von��Willebrand��disease��(vWD)��is��a��common��coagulation��disorder��generally��d
ue��to��a��hereditary��reduction
in��the��quality��or��quantity��of��a��protein��complex��required��for��platelet��a
dhesion,��known��as��von��Willebrand
factor��(vWF).��The��extent��of��deficiency��varies��greatly,��resulting��in��vWD��
subtypes��ranging��from
asymptomatic��to��serious.��A��common��problem��associated��with��vWD��is��menorrha
gia,��and��the��diagnosis
should��always��be��entertained��in��women��who��experience��excessive��menstrual��
blood��loss.��Although��tests
measuring��vWF��are��easily��obtained,��interpretation��of��the��results��can��be��
challenging��since��vWF��levels��can
be��affected��by��blood��type,��inflammation,��infection,��trauma,��and��emotional�
�stress.��Confirmation��of��vWD
often��requires��the��expertise��of��a��hematologist.��Although��the��results��for�
�all��laboratory��tests��listed��can��fall
within��their��reference��ranges��in��a��patient��with��vWD,��the��finding��most��s
uggestive��of��this��diagnosis��is��an
isolated��prolonged��partial��thromboplastin��time.

36

Item��102

This��patient��has��slight��elevations��of��her��alkaline��phosphatase��along��with
��a��deficiency��of��vitamin��D��and
normal��calcium��levels.��This��constellation��of�findings��is��most��consistent��w
ith��secondary��hyperparathyroidism
related��to��vitamin��D��deficiency.��Vitamin��D��storage��is��best��reflected��by�
�the��serum��25-hydroxyvitamin��D
level.��While��there��is��some��disagreement��regarding��normal��levels,��a��level�
�<��10��ng/mL��is��clearly��deficient.
This��would��put��the��patient��at��risk��for��osteomalacia��but��not��rickets,��wh
ich��is��a��clinical��diagnosis��based��on
the��effects��of��insufficient��bone��mineralization��secondary��to��low��vitamin��
D��activity��before��the��closure��of
growth��plates.��Supplementation��with��activated��vitamin��D��(calcitriol)��is��ge
nerally��only��necessary��in��patients
with��renal��failure��or��other��conditions��associated��with��inadequate��activati
on��of��the��storage��forms��of��vitamin
D.��Supplementation��with��vitamin��D��should��decrease��PTH��activity��and��thus��
bone��turnover��in��this��patient,
which��would��likely��normalize��the��alkaline��phosphatase.

Item��103

This��patient��presents��with��a��classic��description��of��torsion��of��the��left�
�testicle,��and��not��an��infectious��process
requiring��antibiotics.��Surgical��exploration��is��the��immediate��priority,��with
��ischemic��damage��starting��between
4��and��8��hours��after��onset.��Attempts��at��manual��detorsion��should��not��dela
y��surgical��exploration��and��often
require��analgesia��or��sedation.��If��surgery��is��not��an��immediate��option,��ma
nual��detorsion��is��performed��by
rotating��the��testicle��from��medial��to��lateral,��like��opening��a��book.��Doppl
er��ultrasonography��is��the��imaging
modality��of�choice,��but��it��delays��surgical��exploration��and��should��be��used
��only��when��the��history��and��physical
examination��make��the��diagnosis��questionable.

Item��104

The��Patient��Protection��and��Affordable��Care��Act��amended��the��Federal��Labor�
�Standards��Act��of��1938��to
require��employers��of��>��50��employees��to��provide��nursing��mothers��reasonable
��break��time��to��express��milk
for��up��to��1��year��following��the��birth��of��their��children.��The��employer��i
s��not��required��to��compensate��nursing
mothers��during��the��breaks��and��is��not��required��to��provide��refrigerated��st
orage��for��the��expressed��milk.��The
law��also��requires��that��the��employer��provide��a��place��to��express��milk,��ot
her��than��a��bathroom,��that��is��shielded
from��view��and��free��from��intrusion��from��coworkers��and��the��public.

37

Item��105

Laboratory��testing��for��Clostridium��difficile��should��be��done��only��on��sympt
omatic��patients.��A��diagnosis��of
C.��difficile��infection��requires��the��presence��of��diarrhea��(>3��unformed��sto
ols��in��a��24-hour��period)��or
radiographic��evidence��of��ileus��and��toxic��megacolon.��In��addition,��the��diag
nosis��requires��a��positive��stool��test
for��toxigenic C. difficile��or��its��toxins,
or��colonoscopic��or��histopathologic��findings��showing
pseudomembranous��colitis.��Laboratory��testing��cannot��distinguish��between��asym
ptomatic��colonization��and
symptomatic��infection.��Test��of�cure��is��not��recommended��after��C.��difficile�
treatment.��Probiotics��may��prevent
antibiotic-
associated��diarrhea��and��may��also��reduce��C.��difficile�diarrhea��in��children�
�and��adults��younger��than
65,��but��are��not��specifically��recommended��for��preventing��or��treating��C.��d
ifficile��infection.

Item��106

Symptoms��consistent��with��proctalgia��fugax��occur��in��13%-
19%��of��the��general��population.��These��consist��of
episodic,��sudden,��sharp��pains��in��the��anorectal��area��lasting��several��secon
ds��to��minutes.��The��diagnosis��is
based��on��a��history��that��fits��the��classic��picture��in��a��patient��with��a��
normal��examination.��All��the��other
diagnoses��listed��would��be��evident��from��the��physical��examination,��except��f
or��sacral��nerve��neuralgia,��which
would��not��be��intermittent��for��years��and��the��pain��would��not��be��transitor
y.

Item��107

This��is��a��cutaneous��horn,��sometimes��referred��to��as��a��hypertrophic��actini
c��keratosis.��It��is��a��horn-like
projection��of�keratin��on��a��slightly��raised��base.��These��usually��arise��in��
areas��subject��to��photoaging,��including
the��forearms.��The��differential��diagnosis��often��includes��keratoacanthoma,��wh
ich��occurs��most��commonly��on
the��face,��grows��very��rapidly,��and��often��is��more��nodular��with��a��central�
�pit.��Pyogenic��granulomas��are��fleshy
appearing,��and��a��wart��or��seborrheic��keratosis��would��both��look��slightly��d
ifferent��but��would��rarely��become
so��large��in��just��months.��Cutaneous��horns��should��be��removed��due��to��the��
possible��development��of��in��situ��or
invasive��squamous��cell��carcinoma��(SOR��A).

38

Item��108

Undertreatment��of��osteoporosis��occurs��frequently��after��a��hip��fracture.��Unl
ess��a��contraindication��exists,
patients��should��be��treated��with��a��bisphosphonate��after��a��hip��fracture,��r
egardless��of��bone��mineral��density
(SOR��C).��Controlled��gastroesophageal��reflux��is��not��a��contraindication��to��
bisphosphonate��therapy.��Proton
pump��inhibitor��use,��but��not��H2-
blocker��use,��is��also��a��modifiable��risk��factor��for��osteoporosis.��Raloxifen
e��and
teriparatide��are��not��bisphosphonates��or��first-
line��therapy��for��prevention��of�recurrent�hip��fracture.��Raloxifene
has��not��been��shown��to��reduce��the��rate��of��nonvertebral��fractures.

Item��109

All��currently��available��influenza��vaccines,��with��the��exceptions��of��recombi
nant��and��cell-culture-based
inactivated��influenza��vaccines,��are��prepared��using��embryonated��egg��culture�
�and��can��potentially��provoke
allergic��and��anaphylactic��reactions.��Large��studies��of�influenza��vaccine��adm
inistration�to��egg-allergic��patients
have��resulted��in��a��few��mild��reactions��but��no��documented��occurrences��of��
anaphylaxis,��although��there��are
reports��of��serious��reactions.��The��data��collected��from��these��studies��provi
ded��sufficient��confidence��for��the
Advisory��Committee��on��Immunization��Practices��to��develop��guidelines��for��adm
inistration��of��influenza
vaccine��in��individuals��with��egg��allergy.

For��those��who��report��that��they��can��eat��lightly��cooked��scrambled��eggs,��v
accination��can��proceed��without
precaution��or��observation.��Those��who��have��experienced��only��hives��can��also
��receive��any��influenza��vaccine
appropriate��for��their��age��and��health��status.��In��the��past��the��CDC��recomm
ended��observing��these��patients��for
30��minutes��afterward,��but��this��recommendation��was��changed��in��2016.��People
��who��have��experienced
symptoms��such��as��hypotension,��wheezing,��nausea,��or��vomiting,��or��reactions�
�requiring��emergency��attention
or��epinephrine��after��eating��eggs��or��egg-
containing��foods��can��also��receive��any��influenza��vaccine��appropriate
for��their��age��and��health��status��and��also��do��not��need��to��be��observed.��
However,��the��vaccine��should��be
administered��by��a��provider��who��can��recognize��and��manage��severe��allergic��
reactions.��Withholding
vaccination��because��of��egg-induced��hives��is��not��recommended.

39

Item��110

Patients��with��low��back��pain��should��be��evaluated��for��the��presence��of��neu
rologic��deficits.��Urinary��retention
is��the��most��frequent��finding��in��cauda��equina��syndrome��(90%��sensitivity),�
�caused��by��compression��of��nerve
roots��from��the��lower��cord��segments.��This��is��usually��due��to��a��massive,��
centrally��herniated��disc,��which��can
result��in��urinary��retention��or��incontinence��from��loss��of��sphincter��functi
on,��bilateral��motor��weakness��of��the
lower��extremities,��and��saddle��anesthesia.��This��problem��should��be��addressed
��urgently.��In��patients��without
urinary��retention,��the��probability��of��the��cauda��equina��syndrome��is��approx
imately��1��in��10,000.

Item��111

This��patient��has��a��low��risk��of��cancer,��based��upon��his��age��and��medical�
�history,��and��no��suggestion��of��a��lower
respiratory��infection.��With��this��presentation,��a��chest��radiograph��is��recom
mended��as��the��first��step��in��the
workup,��and��if��findings��are��normal��he��should��be��observed��for��2-
6��weeks��(SOR��A).��If��there��is��a��recurrence
of��hemoptysis��further��evaluation��is��indicated,��which��should��include��an��in
terval��history,��a��repeat
examination,��and��CT��of��the��chest.

If��the��initial��presentation��had��suggested��a��lower��respiratory��infection,��
treatment��with��oral��antibiotics��rather
than��observation��would��have��been��appropriate.

Item��112

The��Timed��Up��and��Go��test��is��the��most��frequently��recommended��screening��t
est��for��mobility.��It��takes��less
than��a��minute��to��perform��and��involves��asking��the��patient��to��rise��from��
a��chair,��walk��10��feet,��turn,��return��to
the��chair,��and��sit��down.��Any��unsafe��or��ineffective��movement��with��this��t
est��suggests��balance��or��gait
impairment��and��an��increased��risk��of��falling.��If�the��test��is��abnormal,��re
ferral��to��physical��therapy��for��complete
evaluation��and��assessment��should��be��considered.��Other��interventions��should�
�also��be��considered,��such��as��a
medication��review��for��factors��related��to��the��risk��of��falling.

40

Item��113

Hemoglobin��Alc��(HbAlc)��testing��measures��the��percentage��of�glycosylation��of�
the��HbAlc��chain,��and��correlates
to��average��blood��glucose��levels��over��the��previous��2-
3��months.��However,��hypertriglyceridemia,
hyperbilirubinemia,��iron��deficiency��anemia,��splenectomy,��renal��failure,��and�
�aplastic��anemia��can��all��falsely
elevate��HbAlc��levels.��The��other��factors��listed��can��all��falsely��lower��HbA
lc��levels,��as��can��vitamins��C��and��E,
and��acute��blood��loss.

Item��114

The��U.S.��Preventive��Services��Task��Force��has��a��grade��I��recommendation��for
��routine��screening��for
osteoporosis��in��men,��meaning��there��is��insufficient��evidence��to��recommend��
for��or��against��routine��screening.
Men��older��than��50��with��a��minimal-
trauma��fracture��and��men��with��conditions��associated��with��bone��loss��could
be��considered��for��screening.��The��National��Osteoporosis��Foundation��recommend
s��screening��all��men��age��70
and��above��for��osteoporosis.

Item��115

Although��this��patient��has��a��family��history��of��coronary��artery��disease,��s
he��is��under��the��age��of��50��and��thus
aspirin��therapy��as��primary��prevention��is��not��recommended��and��may��increase
��the��risk��for��gastrointestinal
bleeding,��regardless��of��the��dosage.��The��U.S.��Preventive��Services��Task��For
ce��found��insufficient��evidence
for��screening��for��chronic��kidney��disease��even��in��individuals��with��a��posi
tive��family��history.��Neither��a��resting
nor��exercise��EKG��is��recommended��for��asymptomatic��individuals��to��detect��or
��prevent��coronary��artery
disease.

41

Item��116

This��patient��has��signs��and��symptoms��of��acute��salpingitis.��This��condition�
�is��commonly��confused��with
appendicitis,��ectopic��pregnancy,��and��other��pelvic��pathology.��In��this��case�
�the��findings��are��clearly��pelvic��in
origin.��Endocervical��inflammation��with��a��mucopurulent��discharge��is��noted��i
n��almost��every��case��of��acute
salpingitis.��The��acute��nature��of��this��presentation��and��the��adnexal��fullne
ss��suggest��gonorrhea��rather��than
chlamydial��infection,��although��the��antibiotic��regimen��should��probably��cover
��Chlamydia��as��well.��Admission
to��the��hospital��and��treatment��with��parenteral��antibiotics��is��most��appropr
iate��in��this��case��because��of��the
severity��of��the��illness,��the��desire��to��maintain��reproductive��function,��an
d��the��adnexal��fullness.

Routine��laparoscopy��for��every��case��of��salpingitis��is��considered��too��costl
y��and��dangerous.��The��choice��of
intravenous��antibiotic��may��vary,��but��usually��consists��of�a��P-
lactam��antibiotic��(cefoxitin��or��ceftriaxone)��plus
doxycycline,��or��gentamicin��and��clindamycin.��Ceftriaxone,��250��mg��intramuscul
arly,��is��appropriate��for
uncomplicated��gonococcal��infection.

Item��117
Mite��larvae��called��chiggers��cause��itchy��bites.��The��chiggers��crawl��on��ski
n��until��they��reach��constrictive
clothing��like��belts��or��socks��and��then��bite��there.��Flea��bites��are��usuall
y��at��ankle��height��because��fleas��jump.
Mosquito��bites��would��be��diffuse��on��exposed��areas.��Bedbugs��tend��to��bite��
on��the��upper��body��and��neck.��This
is��not��a��typical��presentation��for��Lyme��disease,��which��has��an��initial��ra
sh��that��is��localized��and��not��pruritic.
Item��118

It��is��important��for��health��care��professionals��to��be��familiar��with��the��v
arious��options��for��screening��and��testing
for��latent��or��active��tuberculosis.��In��the��United��States��the��tuberculin��s
kin��test��(TST)��is��the��traditional
screening��test��for��tuberculosis.��The��interferon-
gamma��release��assay��(IGRA)��is��a��blood��test��that��can��also��aid
in��the��diagnosis��of��latent��tuberculosis.��Advantages��of��IGRA��include��the��
ability��to��get��results��without
follow-up��and��the��fact��that��prior��bacille��Calmette-
Guerin��(BCG)��vaccination��does��not��cause��a��false-positive
test.��With��TST��testing,��prior��BCG��vaccination,��especially��if��given��within
��the��last��10��years,��can��result��in
a��false-positive��test.

As��with��the��TST,��conversion��(i.e.,��a��positive��test)��of��IGRA��may��not��oc
cur��within��the��first��8-12��weeks
following��exposure��to��an��individual��with��active��tuberculosis.��In��addition,
��neither��the��TST��nor��the��IGRA��can
distinguish��between��latent��and��active��tuberculosis.

42

There��are��two��IGRA��tests��available��for��use��in��the��United��States��and��bo
th��tests��need��to��be��processed��within
8-
30��hours,��depending��on��the��specific��test��used.��Although��there��is��now��a�
�test��available��that��can��detect
Mycobacterium��tuberculosis��complex��(MTBC)��and��resistance��to��rifampin,��it��i
s��a��separate��test��called��the
Xpert��MTB/RIF��assay.

Item��119

This��patient�s��symptoms��are��consistent��with��the��DSM-
5criteria��for��generalized��anxiety��disorder.��First-line
treatments��for��this��condition��are��SSRIs,��SNRIs,��and��tricyclic��antidepressa
nts.��Quetiapine��and��gabapentin
are��considered��second-
line��medications��for��anxiety��if��control��cannot��be��obtained��with��more��tra
ditional
agents.��Benzodiazepines��such��as��alprazolam��and��clonazepam��are��sometimes��ne
cessary��for��short-term��control
of��anxiety��symptoms��but��are��generally��discouraged��due��to��sedating��side��e
ffects,��the��potential��for��abuse��or
diversion,��and��gradual��tolerance.

Item��120

Bariatric��surgery��has��been��shown��to��reduce��all-
cause��mortality��in��patients��with��morbid��obesity,��mostly��from
reduced��myocardial��infarctions.��Although��orlistat��and��phentermine,��along��wi
th��other��weight��loss��drugs,
have��been��shown��to��be��associated��with��moderate��weight��loss,��there��is��no
��evidence��that��any��of��these��agents
reduce��morbidity��or��mortality.��A��low-
carbohydrate��diet��has��been��associated��with��increased��HDL-cholesterol
levels��and��decreased��triglyceride��levels��when��compared��to��a��low-
fat��diet,��which��may��indicate��a��reduction
in��cardiac��risk.��No��particular��diet��strategy��has��been��shown��to��be��more�
�effective��for��weight��loss��than��any
other��strategy.

43

Item��121

The��combination��of��antipsychotic��medication��and��psychosocial��treatments,��in
cluding��cognitive-behavioral
therapy,��family��therapy,��and��social��skills��training,��is��associated��with��t
he��best��outcomes��in��patients��with
schizophrenia��(SOR��B).��Antipsychotic��medications��should��not��be��combined.��H
ospitalization,��especially��for
the��first��episode��of��schizophrenia,��is��also��recommended��for��the��best��out
come��(SOR��C).��Antidepressant
medication��will��treat��depression��associated��with��schizophrenia��but��will��no
t��necessarily��improve��the
symptoms��of��schizophrenia.

Item��122

Inpatient��video-
electroencephalography��(vEEG)��monitoring��is��the��preferred��test��for��the��dia
gnosis��of
psychogenic��nonepileptic��seizures��(PNES),��and��is��considered��the��gold��stand
ard��(SOR��B).��Video-EEG
monitoring��combines��extended��EEG��monitoring��with��time-
locked��video��acquisition��that��allows��for��analysis
of��clinical��and��electrographic��features��during��a��captured��event.��Many��oth
er��types��of��evidence��have��been
used,��including��the��presence��or��absence��of��self-
injury��and��incontinence,��the��ability��to��induce��seizures��by
suggestion,��psychologic��tests,��and��ambulatory��EEG.��While��useful��in��some��c
ases,��these��alternatives��have
been��found��to��be��insufficient��for��the��diagnosis��of��PNES.

Elevated��postictal��prolactin��levels��(at��least��two��times��the��upper��limit��
of��normal)��have��been��used��to
differentiate��generalized��and��complex��partial��seizures��from��PNES��but��are��
not��reliable��(SOR��B).��While
prolactin��levels��are��often��elevated��after��an��epileptic��seizure,��they��do��
not��always��rise,��and��the��timing��of
measurement��is��crucial,��making��this��a��less��sensitive��test��than��was��previ
ously��believed.��Other��serum��markers
have��also��been��used��to��help��distinguish��PNES��from��epileptic��seizures,��in
cluding��creatine��phosphokinase,
cortisol,��WBC��counts,��lactate��dehydrogenase,��pC02,��and��neuron-
specific��enolase.��These��also��are��not
reliable,��as��threshold��levels��for��abnormality,��sensitivity,��and��specificity
��have��not��been��determined.

MRI��is��not��reliable��because��abnormal��brain��MRIs��have��been��documented��in�
�as��many��as��one-third��of�patients
with��PNES.��In��addition,��patients��with��epileptic��seizures��often��have��norma
l��brain��MRIs.

44

Item��123

In��an��asplenic��patient,��fever��might��be��an��initial��manifestation��of�a��cat
astrophic��infection��and��must��be��treated
immediately��with��a��parenteral��antibiotic��agent.��Quick��administration��of��an
tibiotics��might��prevent��sepsis.
Mortality��can��be��as��high��as��50%��among��patients��with��postsplenectomy��seps
is.��Intravenous��or��intramuscular
ceftriaxone��is��recommended��for��patients��who��have��normal��laboratory��test��r
esults��and��who��do��not��appear
ill.��If��the��patient��lives��more��than��2��hours��from��a��medical��facility��th
at��can��administer��parenteral��antibiotic
therapy,��oral��antibiotics��should��be��given.

Item��124
This��child��has��atypical��Kawasaki��disease.��The��diagnostic��criteria��for��cla
ssic��Kawasaki��disease��is��fever��for
at��least��5��days��and��at��least��four��of��five��principal��clinical��features.�
�The��clinical��features��include:

Changes��of��the��oral��cavity��and��lips:��cracked��erythematous��lips,��strawberr
y��tongue
� Polymorphous��rash:��maculopapular,��erythema��multiforme-
like��or��scarlitiniform��rash,��involving��the
extremities,��trunk,��and��perineal��regions
� Bilateral��nonpurulent��conjunctivitis

Changes��in��the��extremities��(erythema��of��the��hands��and��feet,��desquamation�
�of��the��hands��and��toes��in
weeks��2��and��3)

Cervical��lymphadenopathy��(>��1.5��cm��in��diameter��and��generally��unilateral)

Some��patients��do��not��meet��the��classic��criteria��but��are��labeled��as��havin
g��incomplete��or��atypical��disease.
While��Kawasaki��disease��is��generally��a��clinical��diagnosis��and��there��are��n
o��specific��diagnostic��tests,
supplemental��laboratory��testing��can��help��in��the��diagnosis��of��these��atypic
al��cases.��The��supplemental
laboratory��criteria��include:

� Anemia
� Cerebrospinal��fluid��pleocytosis
� Elevated��C-reactive��protein��and��erythrocyte��sedimentation��rate
� Elevated��liver��enzymes
� Hypoalbuminemia
� Hyponatremia
� Platelets��>��450,000/mm3��after��5��weeks
� Sterile��pyuria
� WBCs��>15,000/mm3

45

Patients��who��have��a��fever��for��5��days��or��more��and��two��or��three��of��the
��classic��criteria��should��be��treated��for
atypical��Kawasaki�s��disease��if��the��C-
reactive��protein��level��is��elevated��and��they��have��three��or��more
associated��laboratory��abnormalities.��The��treatment��of��choice��is��IVIG��and��
high-dose��aspirin��to��reduce��the
risk��of��coronary��abnormalities.��Corticosteroids��have��been��used��as��an��adju
nct��in��refractory��cases��or��with
IVIG,��but��not��alone.

Item��125

This��patient��has��an��acute��migraine��headache��that��did��not��respond��to��NSA
IDs.��Metoclopramide��is��an
effective��treatment��for��migraine��beyond��its��antiemetic��benefit��(SOR��B)��an
d��intravenous��administration��may
be��helpful��for��the��patient��unable��to��tolerate��oral��medications.��Acetamino
phen��has��not��been��proven��to��be
effective��for��migraine.��Opiates��and��barbiturate-
containing��medications��should��only��be��used��for��patients��who
have��failed��multiple��other��treatments��(SOR��C).��Supplemental��oxygen��has��sh
own��efficacy��in��the��treatment
of��cluster��headaches.��Triptan��medications��would��be��another��evidence-
based��choice��for��abortive��therapy��of
migraine.

Item��126

This��patient��presents��with��mild��symptoms��of��hyperandrogenism.��Her��initial�
�laboratory��results��rule��out
pregnancy,��thyroid��disorders,��and��primary��ovarian��failure.��The��follow-
up��laboratory��evaluation��indicates
significant��hyperandrogenism.��The��rapid��onset��and��high��testosterone��level��
suggest��an��ovarian��or��adrenal
tumor��that��should��be��evaluated��by��abdominal/pelvic��imaging.

MRI��of��the��brain��is��not��helpful��in��evaluating��hyperandrogenism.��It��would
��be��appropriate��in��evaluating
hypothalamic��and��pituitary��causes��of��secondary��amenorrhea��such��as��the��fem
ale��athlete��triad��or��other��causes
of��stress��and��malnutrition��that��are��associated��with��weight��loss.��A��karyo
type��would��be��helpful��in��identifying
the��cause��of��primary��amenorrhea.��An��estradiol��level��is��not��helpful��in��e
valuating��hyperandrogenism.��A
dexamethasone��suppression��test��aids��in��the��diagnosis��of��Cushing��syndrome,�
�which��generally��does��not��cause
amenorrhea��and��is��usually��associated��with��stigmata��of��hypercortisolism,��wh
ich��this��patient��does��not��have.

46

Item��127

Recent��studies��suggest��that��financial��exploitation��is��emerging��as��the��mos
t��prevalent��form��of��elder��abuse.
By��the��time��cases��are��detected��the��older��adult�s��financial��resources��hav
e��often��been��drastically��reduced,��so
early��detection��and��intervention��are��critical.��Financial��exploitation��of�ol
der��adults,��which��was��explored��only
minimally��in��previous��studies,��has��recently��been��identified��as��a��virtual�
�epidemic��and��is��a��problem��that��may
be��detected��or��suspected��by��an��alert��physician.

Item��128

The��diagnosis��of��myalgic��encephalomyelitis/chronic��fatigue��syndrome��(ME/CFS)
��is��made��by��determining
that��a��patient�s��symptoms��are��consistent��with��a��standard��definition��of��t
his��disorder��and��by��ruling��out��the
presence��of��other��systemic��diseases��that��can��result��in��similar��symptoms.�
�Currently��suggested��criteria��for
ME/CFS��require��at��least��6��months��of��pathologic��fatigue,��along��with��a��co
mbination��of��other��symptoms
including��post-
exertional��fatigue,��impaired��focus/concentration,��orthostatic��intolerance,��an
d��unrefreshing
sleep.��Other��symptoms��frequently��encountered��in��patients��with��this��conditi
on��include��headache,��diffuse
myalgias,��tender��lymph��nodes,��and��gastrointestinal��or��genitourinary��impairm
ents.��The��diagnosis��of�ME/CFS
should��not��be��considered��in��patients��with��red��flag��symptoms��of��a��potent
ially��more��severe��condition,��such
as��chest��pain,��focal��neurologic��deficits,��joint��erythema/swelling,��enlarged
��lymph��nodes,��or��shortness��of
breath.

Item��129

The��American��Academy��of��Pediatrics��(AAP)��recommends��nebulized��hypertonic��s
aline��for��infants��and
children��hospitalized��with��bronchiolitis��(SOR��B).��They��do��not��recommend��n
ebulized��hypertonic��saline��for
infants��in��the��emergency��department��with��a��diagnosis��of��bronchiolitis.

The��AAP��guideline��also��recommends��that��clinicians��do��not��administer��albut
erol��or��epinephrine��to��infants
and��children��with��a��diagnosis��of��bronchiolitis��(SOR��B).��In��addition,��cli
nicians��should��not��administer
systemic��corticosteroids��to��infants��with��a��diagnosis��of��bronchiolitis��in��
any��setting��(SOR��A).

47

Item��130

Pituitary��adenomas��are��the��most��common��disorder��of��the��pituitary��gland��a
nd��are��responsible��for��10%-15��%
of��all��intracranial��masses.��They��present��with��symptoms��of��hormone��secreti
on��or��a��neurologic��mass��effect,
or��as��an��incidental��finding��on��CT/MRI.��Premenopausal��women��often��experien
ce��amenorrhea,��while��the
most��common��neurologic��symptoms��are��headache��and��vision��changes��(classical
ly��bitemporal��hemianopsia)
due��to��compression��of��the��optic��chiasm.

Tumors��secreting��prolactin��(lactotrophs)��are��the��most��common,��comprising��4
0%-57%��of��all��pituitary
adenomas.��Nonsecreting��tumors��make��up28%-37%,��growth��hormone-
secreting��adenomas��(somatotrophs)
11��%�13%,��and��ACTH-secreting��adenomas��(corticotrophs)��1��%-
2%.��FSH-,��LH-,��and��TSH-secreting��tumors
are��rare.��ACTH-
secreting��tumors��result��in��an��increase��in��circulating��cortisol,��and��the��
diagnosis��is��confirmed
by��24-hour��urine��cortisol,��late-
night��salivary��cortisol,��or��overnight��dexamethasone��suppression��testing.

Gonadotrophs��cause��a��resulting��elevation��of��FSH��and��LH.��Lactotrophs��incre
ase��serum��prolactin,��and
somatotrophs��increase��serum��insulin-
like��growth��factor.��Thyrotrophs��result��in��normal��TSH��with��low��free
t4-

Item��131

Multiple��studies,��including��randomized,��controlled��trials,��have��compared��st
andard��access��to��emergency
contraception��(EC)��with��advance��provision��in��which��the��patient��is��given��
a��prescription��for��the��EC��and
encouraged��to��have��it��filled��in��order��to��have��it��immediately��available��
in��case��of��unprotected��intercourse.��In
multiple��populations��in��the��United��States��and��Europe��advance��provision��co
mpared��to��standard��access��has
been��shown��to��increase��the��rate��of�use��of��EC��and��to��reduce��the��interva
l��between��intercourse��and��use��of��EC.

However,��no��change��has��been��shown��for��several��outcomes��when��advance��prov
ision��was��compared��to
standard��access��to��EC.��These��include��rates��of��sexually��transmitted��infect
ions,��unprotected��intercourse,��use
of��routine��contraceptives��prior��to��sexual��activity,��and��pregnancy��within��
the��population��studied.

48
Item��132

Age-
related��physiologic��differences��such��as��wider��pulse��pressure��should��be��a�
�consideration��when��treating
hypertension��in��the��elderly.��These��patients��also��have��lower��cardiac��outpu
t,��higher��peripheral��resistance,
lower��intravascular��volume,��and��lower��renal��blood��flow��compared��with��youn
ger��patients.��Pulse��pressure
(the��difference��between��systolic��blood��pressure��and��diastolic��blood��pressu
re)��is��a��measure��of��the��degree��of
age-
related��vascular��stiffness��and��is��a��risk��factor��for��coronary��artery��dise
ase��events.

With��increasing��age��the��strongest��predictor��of��coronary��artery��disease��gr
adually��shifts��from��diastolic��blood
pressure��to��systolic��blood��pressure,��and��then��to��pulse��pressure.��Systolic
��blood��pressure��rises��gradually
throughout��adult��life,��whereas��diastolic��blood��pressure��peaks��and��plateaus
��in��late��middle��age,��and��declines
slightly��thereafter.��Diastolic��hypertension��occurs��in��<10%��of��all��patients
��with��hypertension��after��age��70.

Item��133

Asymptomatic��microscopic��hematuria��is��defined��by��the��American��Urological��A
ssociation��(AUA)��as��>3
RBCs/hpf��in��the��absence��of��an��obvious��cause��such��as��menstruation,��infect
ion,��vigorous��exercise,��renal
disease,��trauma,��a��recent��urologic��procedure,��or��a��viral��illness.��Urine��
microscopy��is��required��to��confirm
hematuria��found��on��a��dipstick��examination.

This��patient��has��risk��factors��for��urothelial��cancer,��including��smoking,��h
is��age,��and��his��sex.��In��a��patient��with
no��obvious��cause��for��hematuria,��the��AUA��does��NOT��recommend��repeating��the
��urinalysis��or��treating
empirically��with��antibiotics,��as��this��may��delay��the��diagnosis��of��cancer.�
�In��addition,��assuming��that��benign
prostatic��hyperplasia��(BPH)��is��the��cause��for��his��hematuria��is��inadvisable
;��patients��with��BPH��usually��also
have��risk��factors��for��malignancy.��The��recommended��initial��workup��includes�
�renal��function��testing,��CT
urography,��and��cystoscopy.
49

Item��134

Moderate��evidence��of��lower��risk��from��randomized,��controlled��trials��support
s��the��use��of��low��molecular
weight��heparin��over��unfractionated��heparin��for��treatment��of�venous��thromboe
mbolism��(VTE)��in�pregnancy.
Aspirin��is��not��a��first-
line��treatment��for��VTE,��as��it��crosses��the��placenta��and��has��a��weak��asso
ciation��with
miscarriage.��Clopidogrel��is��not��indicated��for��treatment��of��VTE.��Rivaroxaba
n��and��other��new��non-vitamin
K��oral��anticoagulants��are��not��recommended��because��of��the��lack��of��data��r
egarding��their��use��in��pregnancy.
Warfarin��crosses��the��placenta��and��is��associated��with��fetal��hemorrhage��and
��loss.

Item��135

S-adenosyl-L-
methionine��is��found��in��all��human��cells��and��assists��in��producing��a��wide�
�range��of��compounds
such��as��cartilage��and��neurotransmitters.��It��has��been��shown��to��be��as��eff
ective��as��celecoxib��in��relieving��joint
pain,��but��may��take��up��to��2��months��to��take��effect.��Glucosamine��and��chon
droitin��have��not��been��shown��to��be
effective.��B-
complex��vitamins��may��have��some��benefit��in��diabetic��and��alcoholic��neuropat
hy.��a-Lipoic��acid
and��gamma-
linolenic��acid��have��also��been��shown��to��improve��symptoms��of��neuropathy.

Item��136

This��patient��requires��hospitalization��based��on��her��CURB-
65��score��of��3��(age��>��65,��diastolic��blood��pressure
<��60��mm��Hg,��respiratory��rate��>��30/min,��BUN��<19��mg/dL,��no��confusion),��w
hich��places��her��mortality��risk
at��14%.��Although��azithromycin��has��been��associated��with��an��increased��risk�
�of��myocardial��infarction��in
elderly��patients��hospitalized��with��community-
acquired��pneumonia,��the��combination��of��azithromycin��with
a��P-
lactam�has��been��associated��with��decreased��mortality��in�this��population.��Azi
thromycin��alone��is��acceptable
treatment��in��the��outpatient��setting,��but��not��when��the��patient��requires��h
ospitalization.��A��macrolide��plus��a
P-lactam��antibiotic��has��been��shown��to��have��a��lower��30-
day��mortality��rate��than��a��P-lactam��alone.��This
combination��also��results��in��a��greater��proportion��of�hospitalized��patients��
achieving��clinical��stability��at��7��days
(defined��as��stable��vital��signs��and��oxygen��saturation��>90%��on��room��air)��
when��compared��with��P-lactam
therapy��alone.

50

Item��137

Pertussis��has��been��increasing��in��incidence.��The��initial��presentation��usual
ly��involves��nonspecific��symptoms
including��malaise,��lacrimation,��and��rhinorrhea,��which��is��referred��to��as��t
he��catarrhal��stage.��The��following
stage,��known��as��the��paroxysmal��stage,��is��manifested��by��severe��coughing��t
hat��may��lead��to��the��characteristic
high-
pitched��whooping��sound��when��the��patient��tries��to��catch��his��or��her��breat
h.��Coughing��to��the��point��of
emesis��is��also��characteristic��of��pertussis,��and��the��cough��can��be��severe�
�enough��to��actually��result��in��rib
fractures.��The��cough��may��last��several��weeks��before��it��begins��to��wane��du
ring��the��convalescent��phase,��which
usually��lasts��2-3��weeks.

Pertussis��can��be��diagnosed��clinically��if��there��is��a��coughing��illness��of�
�2��weeks���duration��with��one��of��the
classic��signs��of��pertussis��(post-
tussive��emesis,��respiratory��whoop,��or��paroxysmal��cough)��and��there��is��no
other��apparent��cause.��The��CDC��also��recommends��the��use��of��both��cultures��
and��polymerase��chain��reaction
testing��to��confirm��the��diagnosis.��Culture��is��not��the��best��choice,��howeve
r,��as��it��is��often��done��improperly,
and��culture��results��may��not��be��available��for��several��days.��Polymerase��ch
ain��reaction��has��the��advantage��of
providing��results��in��1-
2��days.��It��has��good��specificity��and��the��sensitivity��is��much��higher��than
��that��of��a
culture;��its��sensitivity��is��highest��during��the��first��2��weeks��of��symptoms
.

Treatment��with��antibiotics��usually��does��not��improve��clinical��symptoms,��alt
hough��it��does��decrease
transmission.��The��CDC��continues��to��recommend��antibiotics��for��pertussis.��Ei
ther��azithromycin��or
clarithromycin��is��currently��recommended.

Ref:

Kline��JM,��Lewis��WD,��Smith��EA,��et��al:��Pertussis:��A��reemerging��infection.�
�Am��Fam��Physician��2013;88(8):507-514.
Item��138

Screening��all��infants��for��developmental��dysplasia��of��the��hip��(DDH)��has��b
een��a��mainstay��of��care��for��many
years.��DDH��is��an��abnormality��of��the��acetabulum��or��femoral��head��and��thei
r��congruence,��presenting��in
infancy.��The��prevalence��of��instability��on��examination��ranges��from��1.6-
28.5��per��1000��births.��While��many
screening��measures��may��be��helpful��in��identifying��DDH,��none��has��been��foun
d��to��improve��long-term��clinical
outcomes��(SOR��C).��The��U.S.��Preventive��Services��Task��Force��found��insuffici
ent��evidence��to��recommend
routine��screening��to��prevent��poor��outcomes.��However,��the��American��Academy�
�of��Pediatrics��and��the
Pediatric��Orthopedic��Society��of��North��America��both��recommend��that��the��phy
sical��examination��of��all
newborns��include��screening��for��DDH.��There��are��no��studies��recommending��the
��abandonment��of��this
screening.

51

Ortolani��(reducing��a��dislocated��hip)��and��Barlow��(dislocating��an��unstable��
hip)��maneuvers��are��commonly
performed��early��in��infancy.��By��2-
3��months��of�age��these��are��less��useful��and��limited��hip��abduction��assessme
nt
is��more��acceptable��(SOR��C).��A��clunk��denotes��a��positive��finding.��Leg��len
gth��measurement��is��useful��in��the
evaluation��of��a��child��presenting��with��a��limp��but��is��not��indicated��in��D
DH.

More��than��50%��of��patients��with��positive��physical��findings��have��been��foun
d��to��have��normal��hips��within��1
month��on��follow-
up��ultrasonography.��Universal��ultrasonography��resulted��in��a��higher��rate��of
��detected��DDH
with��subsequent��treatment��but��did��not��reduce��the��need��for��surgery.��Most�
�of��these��abnormalities��resolve
spontaneously,��and��ultrasonography��should��not��be��used��as��a��universal��scre
ening��measure��(SOR��C).��Plain
films��are��inappropriate��in��infancy��because��the��hip��is��primarily��cartilagi
nous��and��the��associated��radiation
exposure��is��unacceptable.��It��may��be��useful��in��older��children��to��track��p
rogress��after��treatment.

Item��139

The��American��Academy��of��Pediatrics��now��recommends��early��introduction��of��p
eanut-containing��products
for��most��children��to��reduce��the��incidence��of�peanut��allergy.��For��children
��at��high��risk��for��an��allergic��reaction
(e.g.,��those��with��severe��eczema��or��a��first��degree��relative��with��peanut��
allergy)��allergy��testing��can��be
considered��first.��For��this��child,��it��should��be��recommended��that��the��moth
er��introduce��peanut-containing
products��soon.��There��is��no��evidence��that��testing��a��parent��prior��to��intr
oducing��the��child��to��peanut-containing
products��is��beneficial.

Item��140

Smoking��is��the��strongest��modifiable��risk��factor��for��the��development��of��a
n��abdominal��aortic��aneurysm.
Nonmodifiable��risk��factors��include��older��age,��male��sex,��and��a��family��his
tory��of��the��problem.��Other��less
prominent��risk��factors��include��hypertension,��an��elevated��cholesterol��level,
��obesity,��and��preexisting
atherosclerotic��occlusive��disease.

52

Item��141

Prolongation��of��the��QT��interval��is��an��important��medication��adverse��effect
��to��consider.��This��is��particularly
true��in��patients��taking��multiple��medications,��because��this��effect��can��be�
�additive��and��increases��the��risk��of
life-
threatening��arrhythmias��such�as��torsades��de��pointes.��Among��commonly��used��a
ntidepressants,��citalopram
and��escitalopram��may��prolong��the��QT��interval.��Other��SSRIs,��as��well��as��b
upropion,��venlafaxine,��and
mirtazapine,��do��not��have��this��effect.��Both��tricyclic��antidepressants��and��
antipsychotics,��commonly��used��in
patients��also��taking��SSRIs,��can��cause��QT��prolongation,��making��their��combi
ned��use��problematic.

Item��142

Antibiotics��shorten��the��course��of��moderate��to��severe��traveler�s��diarrhea.�
�Azithromycin��is��recommended��as
self-
treatment��for��moderate��to��severe��traveler�s��diarrhea��in��South��and��Southea
st��Asia,��where��Campylobacter
species��are��a��more��common��cause��of��the��illness��than��anywhere��else.��Camp
ylobacter��species��are��resistant
to��fluoroquinolones.��Ciprofloxacin��is��recommended��for��travel��to��South��and�
�Central��America��and��to��Africa.
When��symptoms��are��mild��(1-
3��loose��bowel��movements��per��24��hours��without��limiting��activities),��travel
er�s
diarrhea��can��be��treated��with��loperamide��or��bismuth��subsalicylate.��Rifampin
��and��doxycycline��are��not
commonly��prescribed��for��presumptive��treatment��of��traveler�s��diarrhea.

Item��143

Secondary��hyperparathyroidism��is��most��likely��due��to��low��vitamin��D��intake�
�or��low��serum��vitamin��D,��often
in��the��setting��of�renal��disease.��However,��the��serum��vitamin��D��level��and�
�estimated��glomerular��filtration��rates
are��normal��in��this��patient.��This��rules��out��secondary��hyperparathyroidism,�
�as��well��as��vitamin��D��deficiency
and��renal��insufficiency.��Primary��hyperparathyroidism��is��more��likely��in��the
��presence��of�hypercalcemia��and
inappropriately��normal��or��high��parathyroid��hormone��levels.��Familial��hypocal
ciuric��hypercalcemia��is��ruled
out��by��a��normal��24-hour��urine��calcium��level.

53

Item��144

In��order��to��facilitate��the��most��effective��interview,��the��interpreter��shou
ld��be��as��inconspicuous��as��possible.
This��is��best��achieved��by��having��the��interpreter��seated��next��to��or��sligh
tly��behind��the��patient.

Item��145

Phimosis��is��the��inability��of��the��foreskin��(prepuce)��to��retract��over��the�
�glans��and��it��can��be��physiologic��or
pathologic.��Nonretractile��foreskin��is��very��common��in��young��boys,��and��is��
seen��in��up��to��10%��of
uncircumcised��3-year-
old��boys.��This��physiologic��phimosis��is��part��of�normal��development��and��ove
r��time��the
foreskin��will��become��retractile��due��to��intermittent��erections��and��keratini
zation��of��the��inner��foreskin.
Pathologic��phimosis��is��due��to��distal��scarring��and��on��examination��typicall
y��appears��as��a��white,��contracted
fibrotic��ring��around��the��preputial��outlet.��Pathologic��phimosis,��painful��er
ections��with��a��tight��foreskin,
recurrent��bouts��of��balanitis,��and��recurrent��urinary��tract��infections��in��c
onjunction��with��phimosis��are
indications��for��urologic��consultation��and��consideration��of��circumcision.��Th
is��child�s��examination��is
consistent��with��physiologic��phimosis.

A��short��course��of��a��topical��corticosteroid��(2-
8��weeks��of��0.05%��betamethasone��twice��daily)��applied��to��the
preputial��outlet��may��result��in��accelerated��resolution��of��physiologic��phimo
sis.��However,��given��this��child�s
age��and��the��absence��of�complications,��reassurance��and��continued��good��fores
kin��hygiene��are��recommended.
The��foreskin��should��not��be��forcibly��retracted,��as��this��may��lead��to��micr
otears��and��resultant��scarring.��In��the
absence��of��infection,��neither��antibacterial��nor��antifungal��ointment��is��ind
icated.

Item��146

This��patient��has��sarcoidosis.��Extrapulmonary��manifestations��are��common��in��
patients��with��sarcoidosis��and
erythema��nodosum��is��a��common��cutaneous��sign.��Ocular��symptoms��usually��incl
ude��uveitis,��not
conjunctivitis.��Xerostomia��is��associated��with��Sjogren�s��syndrome.��A��malar��
rash��and��alopecia��are
dermatologic��findings��associated��with��lupus��erythematosus.

54

Item��147

The��treatment��of��diabetes��mellitus��in��frail��elderly��patients,��especially��
nursing��home��residents,��can��be��less
stringent��than��with��other��patients.��Sliding-
scale��insulin��and��diabetic��diets��should��both��be��avoided��in��nursing
home��residents.��Lowering��LDL-
cholesterol��levels��and��aggressive��blood��pressure��control��are��not��indicated
for��frail��elderly��patients.��The��acceptable��levels��of��hemoglobin��Alc��can��
also��be��liberalized,��with��levels��of
8%-9%��being��acceptable.

Item��148

Monoclonal��gammopathy��of�undetermined��significance��(MGUS)��is��present��in��app
roximately��2��%�-3��%��of�the
white��population��older��than��50.��It��is��associated��with��a��risk��of��progres
sion��to��multiple��myeloma��at��a��rate��of
1%��per��year.��Most��patients��diagnosed��with��MGUS��should��be��reevaluated��in�
�6��months��with��a��medical
history,��physical��examination,��CBC,��calcium��and��creatinine��levels,��and��ser
um��electrophoresis,��and��then
annually��thereafter.

Item��149

Isolated��elevated��calcium��levels��should��be��confirmed��before��pursuing��furth
er��testing.��After��calcium
elevation��is��confirmed,��immediate��treatment��should��be��undertaken��if��hyperc
alcemia��is��severe.��Otherwise,
a��history��and��physical��examination��would��be��appropriate,��as��well��as��25-
hydroxyvitamin��D,��magnesium,
creatinine,��and��PTH��levels.��In��patients��with��a��normal��or��elevated��PTH��l
evel,��24-hour��urine��calcium��and
creatinine��levels��can��help��to��differentiate��between��primary��hyperparathyroi
dism��and��familial��hypocalciuric
hypercalcemia.��If��the��PTH��level��is��low,��25-hydroxyvitamin��D,��1,25-
dihydroxyvitamin��D,��and��parathyroid
hormone-
related��peptide��levels��should��be��checked��to��evaluate��possible��causes��of�h
ypercalcemia��independent
of�the��parathyroid.��Sestamibi��scintigraphy��is��indicated��only��after��confirma
tion��of�hyperparathyroidism,��and
typically��in��anticipation��of��surgical��treatment.

55

Item��150

Phalangeal��fractures��of��the��second��to��fifth��toes��can��usually��be��managed�
�with��buddy��taping��and��a��rigid-sole
shoe��for��3��weeks��followed��by��buddy��taping��for��another��3��weeks.��If��the�
�fracture��is��significantly��displaced,
closed��reduction��in��the��office��using��local��anesthesia��may��be��appropriate.

However,��because��of��its��importance��in��weight��bearing��and��balance,��fractur
es��of��the��first��toe��(hallux)��have
a��higher��potential��for��negative��outcomes.��Specifically,��fractures��of�the��h
allux��can��be��managed��nonsurgically
if��they��are��not��displaced��and��involve��less��than��25��
%��of��the��articular��surface��of��the��joint.��Nonsurgical
management��involves��use��of��a��short��leg��walking��cast��with��a��toe��plate��(
extending��past��the��end��of��the��great
toe)��for��3��weeks��and��then��progression��to��a��rigid-
sole��shoe��with��buddy��taping��after��that.

Since��this��patient�s��fracture��involves��more��than��25%��of��the��articular��su
rface��of��the��joint��of��the��first��toe,
early��referral��for��surgical��pinning��by��an��orthopedic��surgeon��would��lead��
to��the��best��outcome.

Item��151

Sedative��hypnotics��such��as��zolpidem��are��associated��with��increased��delirium
��symptoms��and��should��not��be
used��as��first-
line��therapy��(SOR��C).��Behavioral��interventions��such��as��early��and��recurren
t��mobilization,
continuous��observation,��improving��sensory��perception��(such��as��with��hearing�
�aids),��initial��reorientation,��and
removal��of��unnecessary��irritations��such��as��intravenous��lines��and��catheters
��have��been��shown��to��decrease
symptoms��of��delirium��(SOR��B).��While��opioids��can��affect��mental��status,��un
treated��pain��is��also��associated
with��delirium,��and��opioids��are��likely��indicated��in��this��postoperative��pat
ient.��Antipsychotic��medications��such
as��haloperidol,��as��well��as��physical��restraints,��should��be��used��as��last��
resorts��and��only��after��an��appropriate
medical��evaluation��(SOR��A).

Item��152

This��child��has��monosymptomatic��enuresis,��meaning��there��are��no��other��sympt
oms��except��nighttime��bed
wetting.��His��physical��examination��is��normal.��In��this��setting,��the��recomme
nded��initial��workup��is��a��urinalysis
alone.��If��the��results��are��normal��and��there��are��no��other��symptoms��sugges
tive��of��underlying��behavioral��or
medical��conditions,��reassurance��and��instruction��on��possible��behavioral��inte
rventions��is��appropriate��without
further��evaluation.

56
Item��153

The��American��College��of��Physicians��recently��published��clinical��practice��gu
idelines��for��the��treatment��of
pressure��ulcers.��Many��modalities��were��evaluated,��but��good��studies��and��evi
dence��were��available��only��for
a��few��treatments.��Several��good-
quality��studies��indicate��that��protein��supplementation��improves��wound
healing��and��also��appears��to��have��some��benefit��in��prevention��as��well.��Vi
tamin��C��supplementation��made��no
difference��in��wound��healing.��Dextranomer��paste��applied��to��wounds��actually�
�worsened��wound��healing
compared��to��standard��wound��dressings.��Topical��collagen��dressings��and��negat
ive-pressure��wound��therapy
yielded��no��improvement��in��wound��healing��compared��to��usual��care.
Item��154

A��simple��abscess��with��no��overlying��cellulitis��that��is��treated��with��incis
ion��and��drainage��does��not��require
antibiotics.��If��there��were��overlying��cellulitis,��antibiotic��use��would��be��
appropriate.��Purulent��infections��have
a��higher��likelihood��of��being��caused��by��MRSA,��so��coverage��with��trimethopr
im/sulfamethoxazole��or
clindamycin��would��be��appropriate.��In��nonpurulent��skin��infections,��amoxicill
in/clavulanate��or��cephalexin
would��be��appropriate��choices.

Item��155

Pain��in��fibromyalgia��is��thought��to��be��due��to��a��centralized��pain��state.�
�Medications��that��reduce��the��activity
of��neurotransmitters��or��increase��the��activity��of��inhibitory��neurotransmitte
rs��such��as��norepinephrine��and
serotonin��work��best,��and��tricyclic��antidepressants��appear��to��be��most��effe
ctive��(SOR��A).��Older��SSRIs��have
limited��benefit.��Oral��analgesics��only��work��in��about��one-
third��of��patients,��and��opioids��may��increase
hyperalgesia.��Corticosteroids��are��ineffective.

Item��156

Systolic��heart��failure,��or��heart��failure��with��a��reduced��left��ventricular�
�ejection��fraction��(<��40%),��should��be
managed��with��an��ACE��inhibitor��and��a��P-
blocker.��If��there��is��volume��overload,��a��diuretic��should��be��added.
For��those��who��cannot��tolerate��an��ACE��inhibitor,��an��angiotensin��receptor��
blocker��can��be��used.��However,
an��ACE��inhibitor��should��not��be��given��with��an��angiotensin��receptor��blocke
r,��as��this��combination��increases
mortality.

57
Digoxin��will��decrease��symptoms��and��lessen��the��chance��of��hospitalization,��
but��mortality��is��not��reduced.
Mortality��and��hospitalization��can��be��reduced��by��the��addition��of��an��aldos
terone��antagonist��such��as
spironolactone.��In��African-
American��patients,��using��a��combination��of��hydralazine��and��isosorbide��dinit
rate
improves��both��symptoms��and��mortality��for��those��with��New��York��Heart��Assoc
iation��class��III��or��IV��heart
failure��with��a��reduced��ejection��fraction.

Item��157
Healthy��full-term��infants��receive��60�%�-80�
%��of�their��iron��stores��from��their��mothers��during��the��third��trimester
of��pregnancy.��Thus,��even��an��exclusively��breastfed��infant��will��not��typical
ly��be��at��risk��of��developing��iron
deficiency��anemia��until��the��age��of�4��months.��Iron��supplementation��should��
then��be��started��and��continued��until
the��child��is��eating��foods��containing��sufficient��dietary��iron.��Preterm��bab
ies��who��are��exclusively��breastfed
should��begin��iron��supplementation��at��1��month��of��age��due��to��a��foreshorte
ned��third��trimester.

Item��158

Nephrotic��syndrome��is��the��triad��of��edema,��proteinuria,��and��hypoalbuminemia
,��and��it��usually��is��also
associated��with��hyperlipidemia.��The��initial��evaluation��of��this��patient��sho
uld��include��testing��for
hyperlipidemia.��Nephrotic��syndrome��is��associated��with��an��increased��risk��of
��deep��vein��thrombosis��but��does
not��cause��a��coagulopathy.��Diabetes��mellitus��can��be��a��secondary��cause��of�
�nephrotic��syndrome��but��is��not��a
finding��in��primary��nephrotic��syndrome.��Relative��hypocalcemia��may��be��found�
�in��patients��with��nephrotic
syndrome��due��to��low��protein,��but��hypercalcemia��is��not��associated��with��ne
phrotic��syndrome.��Children��and
patients��with��relapses��of��nephrotic��syndrome��have��an��increased��risk��of��i
nfection.

Item��159

Low��vitamin��B12��is��seen��with��prolonged��use��of��metformin,��H2-
blockers,��and��proton��pump��inhibitors.��The
mechanism��is��unclear,��and��it��is��not��known��whether��prophylactic��supplement
ation��prevents��deficiency.
Metformin��reduces��absorption��of��vitamin��B12��in��30%��of��patients��and��reduc
es��vitamin��B12��concentrations��in
5%�10%��of��those��taking��it.��The��dosage��and��duration��of��metformin��therapy�
�is��correlated��with��the��risk��of
vitamin��B12��deficiency.��Given��this��risk,��it��seems��prudent��to��monitor��vit
amin��B12��levels��periodically��in
patients��taking��metformin.

58
Item��160

Clinicians��who��care��for��pregnant��women��need��to��be��familiar��with��the��dia
gnosis��and��monitoring��parameters
for��gestational��diabetes��mellitus,��as��these��help��to��determine��the��need��f
or��management��strategies��outside��of
diet��and��exercise.��The��goal��is��<95��mg/dL��for��fasting��blood��glucose,��<14
0��mg/dL��for��1-hour��postprandial
glucose,��and��<120��mg/dL��for��2-hour��postprandial��glucose.
Item��161

There��is��an��increasing��prevalence��of��hypertension��in��children��and��adolesc
ents.��BMI��is��the��strongest��risk
factor��for��hypertension��in��this��age��group.��Other��risk��factors��include��a�
�family��history��of��hypertension,��low
birth��weight,��male��sex,��and��certain��ethnic��backgrounds.��The��prevalence��of
��hypertension��in��obese��children
is��11%,��which��is��more��than��double��that��of��the��general��pediatric��populat
ion��(reported��at��1��%-5��%).

Item��162

Popliteal��cysts��cause��posterior��knee��pain,��and��are��thought��to��be��a��resu
lt��of��increased��intra-articular��pressure
forcing��fluid��into��the��bursa��and��causing��expansion��and��subsequent��pain.��
Risk��factors��include��rheumatoid
arthritis��or��osteoarthritis��and��advanced��age.��With��the��patient��lying��down
,��a��mass��can��be��palpated��in��the
medial��popliteal��fossa��while��the��knee��is��extended.��When��the��knee��is��fle
xed��to��45���the��pain��subsides��and��the
mass��is��reduced��in��size��or��disappears.

Imaging��is��generally��not��needed��for��the��diagnosis.��Plain��radiographs��are�
�obtained��if��there��is��a��possible
fracture.��Ultrasonography��can��be��used��if��deep��vein��thrombosis��is��a��consi
deration.��If��significant��internal
derangement��of�the��knee��is��suspected,��MRI��may��be��useful.��An��arteriogram��
would��be��an��option��if��aneurysm
of��the��popliteal��artery��were��suspected.

Treatment��is��generally��conservative,��although��addressing��the��underlying��kne
e��pathology��will��prevent��the
cyst��from��recurring.��Pain��is��often��decreased��by��keeping��the��knee��in��fle
xion��and��the��use��of��ice��and��NSAIDs.
If��this��is��not��successful,��intra-
articular��corticosteroids��or��cyst��aspiration��is��often��effective.

59

Item��163

Overscreening��has��become��increasingly��common.��This��inappropriate��care��costs
��the��health��care��system��while
providing��no��benefit��and��possibly��causing��harm��to��patients.��All��of�the��l
isted��screenings��are��appropriate��with
the��exception��of��cervical��cancer��screening��for��a��patient��who��has��had��a�
�hysterectomy.��The��U.S.��Preventive
Services��Task��Force��recommends��against��screening��for��cervical��cancer��in��w
omen��over��65��who��have��had
adequate��recent��screening,��and��in��women��who��have��had��a��hysterectomy��for�
�benign��disease��(SOR��A).

Item��164

Evidence-
based��guidelines��for��the��treatment��of��patients��with��acute��coronary��syndro
me��support��several
medications��in��the��subacute��period.��Dual��antiplatelet��therapy,��such��as��cl
opidogrel��combined��with��aspirin,
has��been��shown��to��reduce��cardiovascular��mortality��(SOR��B).��ACE��inhibitors
��and��statins��should��be��initiated
immediately��after��a��myocardial��infarction��and��continued��indefinitely��to��re
duce��mortality��and��the��risk��of
repeat��infarction��(SOR��A).��P-
Blockers��have��been��shown��to��improve��mortality��in��patients��with��a��left
ventricular��ejection��fraction��<40%��(SOR��A).��Nitroglycerin��is��often��used��t
o��manage��angina��but��has��no
demonstrated��mortality��benefit.

Item��165

Somatic��disorders��usually��require��a��multifaceted��approach��to��treatment.��It
��is��important��to��schedule��regular
visits��at��short��intervals��to��establish��a��collaborative��relationship��with��
the��patient.��It��is��also��important��to��limit
diagnostic��testing��and��reassure��the��patient��that��serious��diseases��have��al
ready��been��ruled��out,��and��to��screen
for��other��mental��illnesses.��Referral��for��counseling��using��cognitive-
behavioral��therapy��and��mindfulness-based
therapies��is��also��effective.��SSRIs��and��tricyclic��antidepressants��have��been
��found��to��be��the��most��effective
pharmacotherapy��(SOR��B)��for��somatic��symptom��disorders.��Amitripityline��is��t
he��most��studied��tricyclic
antidepressant,��and��trials��have��shown��that��it��has��a��greater��likelihood��o
f��success��compared��to��SSRIs.��Other
antidepressants,��anticonvulsants,��and��antipsychotic��medications��are��ineffecti
ve��and��should��be��avoided��(SOR
B).

60

Item��166

This��case��can��be��best��described��as��acute��lumbar��pain��with��radiculopathy.
��There��are��no��red��flags��that��would
suggest��the��presence��of��a��time-
sensitive��condition��such��as��cauda��equina��syndrome,��cancer,��infection,��or
fracture.��In��cases��like��this,��consensus��agreements��recommend��against��any��
imaging��studies.��Instead,��a
6-
week��course��of��pain��medication,��manipulation,��physical��therapy,��and��self-
care��should��be��undertaken.

Item��167

Nausea��is��a��common��symptom��near��the��end��of��life��and��can��have��many��cau
ses.��One��of��the��first��objectives
in��the��palliative��care��of��nausea��is��to��identify��and��address��the��underly
ing��cause.��This��patient��is��taking��a
long-
acting��opiate��that��predisposes��her��to��constipation.��Furthermore,��the��famil
y��seems��to��accept��the��fact
that��she��is��not��having��any��bowel��movements��because��she��is��not��eating��v
ery��much,��which��is��an��incorrect
understanding.��There��are��no��signs��of��dehydration��to��warrant��subcutaneous��
hydration,��nor��are��there��signs
of��bowel��obstruction��that��would��warrant��a��nasogastric��tube.

Item��168

Frailty��is��an��age-
related,��multidimensional��state��of��decreased��physiologic��reserves.��Frail��pa
tients��are��at
increased��risk��of��decline��as��a��result��of��illness��or��stressors��such��as��
surgery.��The��definition��and��assessment
of��frailty��are��still��not��uniformly��agreed��upon,��but��gait��speed��is��recog
nized��as��a��highly��reliable��single
measurement��tool.��The��other��options��listed��are��useful��components��of�a��ger
iatric��assessment��but��are��not��used
as��a��single��test��for��the��evaluation��of��frailty.

61

Item��169

Benzoyl��peroxide��is��the��only��topical��antibiotic��without��evidence��to��sugge
st��that��its��use��contributes��to
bacterial��antimicrobial��resistance.��For��this��reason��it��is��recommended��as��
first-line��therapy��in��mild��to
moderate��inflammatory��acne��and��as��an��adjunctive��agent��with��other��topical�
�antibiotics��that��can��induce
bacterial��resistance,��such��as��clindamycin��and��erythromycin.��Dapsone��is��an�
�antibiotic��but��is��thought��to
primarily��improve��acne��via��its��anti-
inflammatory��effect.��Metronidazole��is��used��chiefly��in��acne��rosacea
rather��than��acne��vulgaris��and��it��may��contribute��to��antimicrobial��resistan
ce.

Item��170

Peppermint��oil��is��an��antispasmodic��that��has��moderate��evidence��of��benefit�
�in��the��relief��of��abdominal
cramping��in��patients��with��irritable��bowel��syndrome��(IBS).��Probiotics��as��a
��group��have��some��evidence��of
benefit��in��improving��global��IBS��symptoms,��but��the��studies��have��used��diff
erent��probiotic��preparations��so��it
is��difficult��to��draw��definite��conclusions��about��their��effectiveness��or��to
��recommend��a��specific��preparation.
Polyethylene��glycol��is��an��osmotic��laxative��that��has��evidence��of��benefit��
for��bowel��symptoms,��including
improvement��in��stool��frequency��and��consistency,��but��it��does��not��improve��
abdominal��pain.��Wheat��bran��is
an��insoluble��fiber��that��contains��fructans��that��can��exacerbate��IBS��symptom
s��and��should��be��avoided��in��patients
with��IBS.

Ref:

Chey��WD,��Kurlander��J,��Eswaran��S:��Irritable��bowel��syndrome:��A��clinical��re
view.��JAMA��2015;313(9):949-958.

Item��171

The��American��Academy��of��Pediatrics��recommends��that��medical��care��for��child
ren��with��an��urgent��or
emergent��condition��never��be��withheld��or��delayed��because��of��problems��obtai
ning��consent.��All��efforts��to
obtain��consent��should��be��documented.��In��the��case��of��a��4-year-
old��male��who��is��suspected��to��have��meningitis,
even��a��brief��delay��in��treatment��could��result��in��a��bad��outcome��for��the�
�patient.

62
Item��172

This��patient��presents��with��the��classic��symptoms��and��signs��of��rosacea.��FD
A-approved��first-line��agents��for
treating��rosacea��include��metronidazole��0.75%��lotion,��0.75%��cream,��and��1%��
gel;��azelaic��acid��15%��gel;
sulfacetamide��10%/sulfur��5��
%��cream,��foam,��lotion,��and��suspension;��brimonidine��0.33��
%��gel;��and��ivermectin
1%��cream��(SOR��A).

Item��173

Bulimia��nervosa��is��an��eating��disorder��associated��with��recurrent��binge��eat
ing��and��induced��vomiting.��There
is��also��a��history��of�excessive��physical��activity��in��many��cases.��Because��
of��vomiting,��the��teeth��are��stained��and
there��is��destruction��of��the��enamel��from��stomach��acid.��Parotid��swelling��i
s��also��noted��frequently.��With
mumps��and��bacterial��parotitis��the��patient��feels��sick��and��parotid��glands��
are��tender.��Cat��scratch��disease��and
mononucleosis��affect��lymph��glands.

Item��174

A��new��treatment��may��not��be��more��effective��than��available��treatments��but�
�it��may��have��lower��costs,��fewer
adverse��effects,��or��greater��convenience��compared��to��standard��treatment.��A�
�treatment��with��other��advantages
might��be��preferred��to��standard��treatment��for��some��patients.��A��non-
inferiority��trial��rigorously��evaluates��a
treatment��against��accepted��and��effective��treatments,��with��the��goal��of��dem
onstrating��that��it��is��at��least��almost
as��good.��Single-��and��double-
blind��studies��(with��or��without��crossover��of��the��control��and��experimental
groups)��can��be��done��to��demonstrate��the��drug��is��effective,��but��showing��t
hat��a��new��treatment��is��not��inferior
to��standard��treatments��can��be��rigorously��demonstrated��by��a��non-
inferiority��trial.��A��non-inferiority��trial��does
not��distinguish��between��a��new��treatment��that��is��not��inferior��and��one��th
at��is��truly��superior,��and��cannot
demonstrate��equivalence.��A��case-
control��trial��is��not��appropriate��to��compare��one��medication��to��another;��i
ts
retrospective��design��is��better��for��evaluating��risk��factors��or��exposures��a
s��possible��causes��or��contributors��to
a��disease��or��illness,��as��it��involves��no��intervention��in��either��the��cont
rol��or��the��experimental��group.

63
Item��175

Streptococcus��species��and��Staphylococcus��aureus��are��the��most��common��causes
��of��skin��and��soft-tissue
infections.��With��a��puncture��wound��caused��by��an��object��penetrating��the��so
le��of��the��shoe,��another��organism
that��should��be��considered��as��a��possible��cause��of��the��infection��is��Pseud
omonas��aeruginosa.��Klebsiella��is��not
a��common��cause��of��these��infections.��Pasteurella��multocida��is��a��common��ca
use��of��infection��from��a��cat��bite,
and��can��also��be��seen��in��dog��bites.��Corynebacterium��is��also��associated��w
ith��infections��from��animal��bites.

Vibrio��vulnificus��should��be��considered��when��there��is��a��skin��infection��af
ter��exposure��to��salt��water.

Item��176

If��isolated��ear��anomalies��such��as��cup��ears,��preauricular��skin��tags,��or��
ear��pits��are��associated��with��maternal
gestational��diabetes��mellitus,��ultrasonography��is��warranted��to��screen��for��
renal��abnormalities.��Ear
malformations��are��also��associated��with��renal��abnormalities��in��children��wit
h��other��malformations,
dysmorphic��features,��a��family��history��of��deafness,��or��teratogenic��exposure
s��(SOR��C).

Item��177

According��to��the��U.S.��Preventive��Services��Task��Force,��screening��for��major
��depression��is��recommended
if��adequate��treatment��and��follow-
up��can��be��provided.��Sexually��active��females��should��be��screened��for
Chlamydia��infection.��Cervical��cancer��screening��begins��at��age��21.��Because��
of�undue��harm��from��unnecessary
treatment,��scoliosis��screening��is��not��recommended.��There��is��insufficient��e
vidence��to��recommend��for��or
against��routine��screening��for��alcohol��use��(SOR��A,��B).

Ref:

Ham��P,��Allen��C:��Adolescent��health��screening��and��counseling.��Am��Fam��Physi
cian��2012;86(�12):�1109-1116.

Item��178

Depression��can��be��treated��with��counseling��and/or��medication.��The��choice��o
f��medication��depends��on��many
factors��including��side��effects,��dosing��schedule,��cost,��patient��preference,�
�and��comorbidities.��In��a��patient��with
significant��depression��and��no��comorbidities,��a��second-
generation��SSRI��should��be��the��initial��medication
choice.��Fluoxetine��is��the��only��SSRI��choice��listed.

64

Duloxetine��is��an��SNRI��that��may��lead��to��sweating,��tachycardia,��urinary��re
tention,��and��elevation��of��blood
pressure.��It��is��indicated��if��the��patient��has��fibromyalgia.��Trazodone��is��
associated��with��somnolence,
orthostatic��hypotension,��and��priapism.��It��may��be��used��in��low��doses��as��a
n��adjuvant��in��patients��with��insomnia.
Mirtazapine��can��cause��sedation,��weight��gain,��increased��appetite,��dry��mouth
,��dizziness,��and��constipation.
It��is��helpful��for��patients��with��significant��insomnia��and��loss��of��appetit
e.��Amitriptyline��is��a��tricyclic
antidepressant��and��has��significant��anticholinergic��side��effects.��It��is��als
o��associated��with��conduction
abnormalities��that��can��lead��to��arrhythmias.
Item��179

An��intertriginous��rash��of��brownish-
red��macules��that��often��coalesce��is��characteristic��of��a��bacterial��infectio
n
caused��by��Corynebacterium��minutissimum��known��as��cutaneous��erythrasma.��Coral
-red��fluorescence��under
a��Wood�s��lamp��confirms��the��diagnosis.��The��best��treatment��for��erythrasma��
is��oral��or��topical��erythromycin,
with��the��oral��form��being��most��effective.��Topical��clindamycin��and��antibact
erial��soaps��may��also��be��of��some
benefit.

Item��180

Flow��cytometry��and��immunophenotyping��of��peripheral��blood��demonstrate��clonal
ity��of��the��circulating
lymphocytes��in��chronic��lymphocytic��leukemia��(CLL).��Absolute��lymphocytosis��i
s��defined��as��>5000
cells/mm3.

Ninety��percent��of��CLL��patients��are��asymptomatic��at��the��time��of��diagnosis
.��Imaging��procedures��are��not
necessary��for��establishing��the��diagnosis.��While��a��bone��marrow��biopsy��or��
aspiration��and/or��a��lymph��node
biopsy��may��be��necessary��as��the��disease��progresses,��they��are��not��routinel
y��needed��to��establish��a��CLL
diagnosis.
65

Item��181

Illicit��substance��use��is��the��most��common��medical��cause��or��secondary��caus
e��of��acute��psychosis.��Conditions
diagnosed��by��brain��imaging,��blood��chemistry,��and��HIV��tests��(intracranial��
mass,��paraneoplastic��syndrome,
and��HIV��infection,��respectively)��are��less��common��medical��causes��of�acute��
psychosis.��Major��depression��with
psychosis��would��be��considered��a��psychiatric��or��primary��cause��of��psychosis
.

Item��182

In��the��evaluation��of��a��solitary��pulmonary��nodule,��a��risk��assessment��is��
used��to��determine��the��diagnostic
evaluation��of��the��nodule.��Clinical��risk��factors��for��malignancy��include��ol
der��age,��a��history��of��smoking,��and
a��previous��history��of��cancer.��Radiologic��features��indicative��of��malignancy
��include��a��nodule��diameter��>��10
mm,��an��upper��lung��location,��irregular��or��spiculated��borders,��no��calcifica
tions,��a��ground-glass��appearance,
and��increasing��size��over��time.��Radiologic��characteristics��suggesting��benign
��disease��include��a��size��<��5��mm,
central��calcifications,��smooth��borders,��a��solid��appearance,��and��a��stable��
size��over��1��year.

Item��183

Rapid��administration��of�systemic��corticosteroids��can��reduce��the��frequency��o
f�hospitalization��in��patients��with
severe��asthma��who��present��with��an��exacerbation��(SOR��A).��There��is��no��cle
ar��benefit��from��parenteral��versus
oral��administration��(SOR��B).��The��same��benefit��is��not��seen��with��inhaled��
corticosteroids��during��an
exacerbation.��Montelukast��and��cromolyn��are��not��appropriate��treatments��for��
an��asthma��exacerbation.
Nebulized��epinephrine��is��used��in��the��treatment��of��croup.
66

Item��184

The��safest��and��most��effective��treatment��for��postmenopausal��atrophic��vagini
tis��is��local��estrogen��such��as��the
estradiol��vaginal��ring,��estradiol��vaginal��tablets,��or��estrogen��cream.��All�
�of�these��therapies��are��effective,��and
while��there��is��a��theoretical��risk��of��small��amounts��of��systemic��absorptio
n,��there��is��no��evidence��to��suggest
that��there��is��an��increased��risk��of��endometrial��cancer��or��thromboembolic��
disease��related��to��this��type��of
estrogen��use.��These��therapies��are��therefore��preferred��over��systemic��estrog
ens��such��as��estradiol��or��a
combination��of��conjugated��estrogens��and��medroxyprogesterone.��Raloxifene,��a��
synthetic��estrogen
agonist/antagonist,��has��not��been��found��to��be��effective��for��the��treatment�
�of��atrophic��vaginitis.��The��vaginal
ring��is��approved��for��use��as��a��contraceptive��and��is��not��intended��for��us
e��in��postmenopausal��patients.

Item��185

Down��syndrome��is��the��most��common��chromosomal��abnormality.��The��5-
year��survival��rate��is��>90%,��with
most��individuals��living��past��60��years��of��age,��and��there��has��been��signif
icant��progress��in��improving��quality
of��life��for��these��patients.��Infants��with��Down��syndrome��present��with��a��w
ide��variety��of��symptoms��and��signs,
however,��and��there��are��certain��health��complications��that��can��increase��the
��risk��of��hospitalization.

All��infants��with��Down��syndrome��should��be��screened��for��cardiac,��feeding,��
vision,��hearing,��thyroid,��and
hematologic��abnormalities.��Up��to��50%��manifest��congenital��cardiac��defects,��
and��a��cardiology��consultation
and��echocardiography��are��warranted.��Hypotonia��may��affect��feeding��and��shoul
d��be��noted��for��assessment.
Congenital��cataracts��are��common,��so��a��red��reflex��examination��is��imperativ
e.��Cervical��spine��films,��hip
ultrasonography,��and��polysomnography��are��not��recommended��in��a��newborn��with
��Down��syndrome.
However,��all��children��with��Down��syndrome��should��have��polysomnography��betwe
en��the��ages��of��1��and��4
years.��Due��to��concerns��about��atlantoaxial��instability��parents��should��be��a
dvised��to��monitor��for��any��signs��of
spinal��cord��impingement.

Screening��for��subclinical��thyroid��disease��should��also��be��performed,��as��we
ll��as��a��CBC��to��evaluate��for
transient��myeloproliferative��disorder,��leukemoid��reaction,��and��polycythemia,�
�which��all��occur��more
frequently��in��Down��syndrome��patients.

It��is��important��to��congratulate��the��parents��on��their��newborn��and��careful
ly��explain��the��rationale��for��each
recommended��test��and��discuss��results��and��their��meaning��clearly.��An��Indivi
dual��Family��Service��Plan��is��also
recommended,��with��a��review��every��6��months��to��evaluate��specific��measures��
indicated��for��each��age��group.

67

Item��186

Evidence��strongly��supports��the��use��of��dual��antiplatelet��therapy��with��aspi
rin��and��a��P2Y12��inhibitor��such��as
clopidogrel��for��a��minimum��of��6-12��months��in��patients��with��a��drug-
eluting��stent.��Aspirin��at��a��dosage��of��81
mg��is��as��effective��as��325��mg,��if��not��more��effective.��It��is��also��assoc
iated��with��a��lower��bleeding��risk.
Prasugrel��is��a��second-
line��antiplatelet��agent��and��can��be��used��when��there��is��a��contraindication
��to��clopidogrel
or��if��the��patient��is��resistant��to��clopidogrel.��When��it��is��used,��it��sho
uld��be��used��with��an��aspirin��dosage��of��81
mg��daily.��Aspirin/dipyridamole��is��effective��for��secondary��stroke��prevention
��but��does��not��have��a��role��in��the
treatment��of��ischemic��cardiovascular��disease.

Item��187

Secondary��centrally��mediated��hypothyroidism��is��caused��by��hypothalamic��or��p
ituitary��disease��in��which��TSH
is��not��being��produced,��resulting��in��failure��to��stimulate��the��thyroid��gla
nd��to��make��thyroid��hormone.��Standard
treatment��for��this��type��of�hypothyroidism��is��levothyroxine,��but��determining
��the��response��to��therapy��requires
measuring��free��T4��rather��than��TSH.��Desiccated��thyroid��USP��is��a��combinati
on��of��T3��and��T4��and��response
to��treatment��is��usually��assessed��with��TSH��levels.��T3��levels��can��fluctuat
e��in��patients��treated��with��desiccated
thyroid,��as��well��as��those��who��are��on��liothyronine��(T3),��and��are��therefo
re��generally��not��used.��TSH��is��also
used��to��monitor��treatment��in��patients��with��surgical��hypothyroidism.��Graves
��disease��is��an��autoimmune
disorder��commonly��causing��hyperthyroidism,��but�patients��usually��become��hypot
hyroid��after��treatment.��TSH
is��used��to��monitor��these��patients��as��well.

Item��188

This��patient��has��severe��hyperkalemia��and��needs��urgent��treatment.��Intraveno
us��insulin��followed��by��glucose
will��shift��potassium��intracellularly��and��is��an��effective��treatment��for��se
vere��hyperkalemia��(SOR��B).��Sodium
polystyrene��sulfonate��is��not��recommended��as��an��urgent��treatment��and��would
��not��be��an��initial��treatment��for
severe��hyperkalemia.��Intravenous��calcium��gluconate��solution��does��not��lower�
�serum��potassium��but��is
indicated��to��prevent��arrhythmias��in��patients��with��hyperkalemia��and��EKG��ch
anges.��Intravenous��furosemide
is��not��a��treatment��for��hyperkalemia,��although��hypokalemia��is��a��common��si
de��effect.��Hemodialysis��is��a
treatment��for��severe��hyperkalemia��but��is��not��considered��a��first-
line��treatment.

68
Item��189

Croup��is��a��common,��self-
limited��illness��caused��by��viral��infection��of��the��upper��respiratory��tract.
��The
diagnosis��is��based��primarily��on��the��clinical��history��and��examination��find
ings.��A��history��of��the��abrupt��onset
of��a��barking��cough,��inspiratory��stridor,��and��hoarseness��in��a��2-year-
old��child��is��typical��of��croup,��although
it��can��present��at��any��age��between��6��months��and��12��years.��Low-
grade��fever,��a��barking��cough,��and��varying
degrees��of��respiratory��distress��(nasal��flaring,��retractions,��or��stridor)��a
re��typically��present��on��examination.
Findings��such��as��a��toxic��appearance,��wheezing,��drooling,��and��difficulty��s
wallowing��are��not��consistent��with
the��diagnosis.

Treatment��of��mild��disease��with��a��corticosteroid��has��proven��benefit��(SOR��
A)��even��when��administered��as��a
single��oral��dose��(SOR��B).��Nebulized��epinephrine��has��been��shown��to��improv
e��outcomes��in��children��with
moderate��to��severe��croup��(SOR��A).��Humidification��therapy��in��the��emergency
��department��setting��provides
no��benefit��(SOR��A).��This��child��has��mild��croup��and��a��single��dose��of��de
xamethasone��(0.15-0.60��mg/kg,
usually��given��orally)��followed��by��close��observation��is��the��most��appropria
te��treatment.

Item��190

Intensive��glycemic��control��in��type��2��diabetes��mellitus��results��in��a��redu
ced��rate��of��progression��to
macroalbuminuria,��which��is��an��indicator��of��the��development��of��diabetic��ne
phropathy.��Intensive��glycemic
control��does��not��reduce��the��likelihood��of��dialysis,��nor��does��it��improve�
�blood��pressure��control.��Intensive
control��does��increase��the��incidence��of�hypoglycemic��events,��which��is��parti
cularly��hazardous��in��the��elderly.
As��was��demonstrated��in��the��Action��to��Control��Cardiovascular��Risk��in��Diab
etes��(ACCORD)��trial,��intensive
glycemic��control��actually��increases��all-cause��mortality.

Item��191

Porphyria��cutanea��tarda��(PCT)��should��be��suspected��in��patients��who��present
��with��blistering��lesions��on
sun-
exposed��skin.��Factors��associated��with��increased��susceptibility��to��PCT��incl
ude��those��that��cause��liver
damage,��such��as��alcohol��use��and��hepatitis��C,��as��well��as��smoking,��estrog
en��use,��HIV��infection,��and��HFE
mutations.��The��initial��test��in��suspected��PCT��should��be��total��serum,��plas
ma,��or��spot��urine��porphyrins.��The
clinical��presentation��of��this��patient��is��less��consistent��with��cellulitis,�
�which��usually��presents��with��erythema,
pain,��and��warmth,��and��when��severe��may��lead��to��fever��and��leukocytosis.

69
Item��192

Psychostimulants��are��the��most��effective��medication��option��for��treating��att
ention-deficit/hyperactivity
disorder��(ADHD)��and��are��widely��accepted��as��first-
line��therapy.��Behavioral��therapy��is��recommended��before
considering��medications��for��children��under��the��age��of��6,��and��may��be��con
sidered��for��older��children��but��may
not��add��much��benefit��in��this��age��group,��particularly��when��there��is��a��g
ood��response��to��medication.
Atomoxetine��and��the��a2-receptor��agonist��guanfacine��are��considered��second-
line��agents��because��of��lower
effectiveness��against��core��ADHD��symptoms��and��a��greater��frequency��of��side�
�effects.��Bupropion��has��been
used��off-label��for��ADHD��but��is��not��FDA��approved��for��this��indication.

Item��193

This��patient��has��ruptured��the��flexor��digitorum��profundus��(FDP)��tendon��as�
�a��result��of��a��sudden��force��applied
to��the��distal��phalanx��while��in��flexion.��This��happens��when��catching��the��
finger��in��an��opponent�s��jersey��or��belt
loop��while��making��a��tackle,��hence��the��name��jersey��finger.��This��injury��i
s��also��seen��in��wrestlers.

The��FDP��attaches��at��the��base��of��the��distal��phalanx��while��the��flexor��di
gitorum��sublimis��(FDS)��attaches��at
the��base��of��the��middle��phalanx.��Thus,��it��can��be��determined��by��physical�
�examination��which��tendon��is
injured,��providing��for��early��and��accurate��diagnosis��and��proper��treatment.�
�With��the��noninvolved��fingers��held
in��full��extension,��if��the��patient��can��flex��the��proximal��interphalangeal��
(PIP)��joint��the��FDS��is��intact.��With
rupture��of��the��FDP,��distal��interphalangeal��(DIP)��joint��flexion��may��still�
�be��weakly��present,��but��the��patient
is��unable��to��hold��flexion��against��resistance��and��this��maneuver��elicits��p
ain.��With��rupture��of��both��flexor
tendons,��the��patient��is��unable��to��flex��either��the��PIP��or��the��DIP��joint
.��In��the��scenario��presented,��rupture��of
both��flexor��tendons��is��unlikely.��Rupture��of��the��extensor��digitorum��tendon
s��may��cause��mallet��finger��at��the
DIP��joint,��or��a��boutonniere��deformity��of��the��PIP��joint.��Since��many��of��
these��flexor��tendon��injuries��require
surgical��exploration��and��repair,��early��orthopedic��consultation��is��recommend
ed.

Item��194

Anticoagulant��therapy��is��the��treatment��for��pulmonary��embolism��and��should��
be��started��immediately.��A
rapid-
acting��parenteral��anticoagulant��(heparin,��enoxaparin)��or��a��direct��orally��a
ctive��factor��Xa��inhibitor
(rivaroxaban)��should��be��started��initially.��After��the��initial��treatment,��th
e��factor��Xa��inhibitor��should��be
continued��or��the��parenteral��anticoagulant��should��be��transitioned��to��an��or
al��anticoagulant.��Patients��who
develop��a��pulmonary��embolism��as��a��complication��of��a��reversible��factor��ha
ve��a��low��risk��of��recurrence��when
the��anticoagulant��therapy��is��stopped.��A��3-
month��course��of��anticoagulant��is��adequate��treatment.

70

Item��195

Proper��sleep��position��is��highly��important��in��reducing��the��risk��of��sudden
��infant��death��syndrome��(SIDS).��Side
sleeping��and��prone��sleeping��increase��the��risk��for��SIDS��(SOR��A).��Bed��sha
ring��also��increases��the��risk.��The
�Back��to��Sleep���campaign��of��the��American��Academy��of��Pediatrics��(AAP)��was
��associated��with��a��53%
decrease��in��the��incidence��of��SIDS��in��the��United��States��in��the��1990s.

The��use��of��pacifiers��during��sleep��decreases��the��risk��for��SIDS,��as��does�
�exclusive��breastfeeding.��The��AAP
recommends��offering��a��pacifier��for��use��during��sleep��for��infants��in��whom�
�breastfeeding��is��well��established.

There��are��also��multiple��invariable��risk��factors��for��SIDS.��These��include��
male��sex,��Native��American
ethnicity,��birth��weight��<��2500��g,��and��birth��before��37��weeks��gestation.��
Large��for��gestational��age��infants��are
not��at��increased��risk��for��SIDS,��and��infants��of��Asian��or��Pacific��Islande
r��ethnicity��actually��have��a��lower��than
average��risk��of��SIDS.��Apparent��life-
threatening��events��are��not��a��risk��factor��for��SIDS��and��are��not��related
to��SIDS��(SOR��B)..
Item��196

BNP��is��a��sensitive��marker��for��heart��failure��and��procalcitonin��is��a��reas
onable��marker��for��acute��infection.
Serum��lactate��is��used��to��help��assess��the��severity��of�hypoperfusion,��or��s
hock.��The��erythrocyte��sedimentation
rate��and��C-
reactive��protein��level��are��not��specific��and��would��not��be��good��discrimina
tors��in��this��case.��Serial
troponin��and��creatine��phosphokinase��can��help��rule��out��acute��myocardial��is
chemia��but��would��not��help
discriminate��between��heart��failure��and��pneumonia.��D-
dimer��is��a��marker��for��thromboembolism��and��is��not
relevant��in��this��situation.

71

Item��197

The��diagnosis��and��treatment��of��left��ventricular��hypertrophy��(LVH)��in��pati
ents��with��hypertension��is
important,��as��it��is��associated��with��increased��morbidity��and��mortality��fro
m��heart��failure,��arrhythmias,��and
sudden��cardiac��death.��LVH��is��often��seen��in��patients��with��long-
standing��uncontrolled��hypertension.��If��the
sum��of��the��S��wave��in��lead��VI��and��the��R��wave��in��lead��V5��or��V6��is��>
35��mm��or��the��R��wave��in��aVL��is��>11
mm,��it��suggests��the��presence��of��LVH��(Sokolow-
Lyon��indices).��The��sensitivity��of��an��EKG��for��LVH��ranges
from��7%��to��50%,��however,��so��echocardiography��is��the��test��of��choice��to��
make��the��diagnosis.
Echocardiography��helps��to��directly��visualize��and��quantify��left��ventricle��w
all��size,��in��addition��to��providing
other��valuable��information��such��as��cardiac��wall��motion��abnormalities,��ejec
tion��fraction,��and��cardiac
chamber��dimensions.��There��is��no��primary��role��for��the��other��tests��listed�
�in��this��setting��(SOR��B).

Item��198

This��patient��has��gestational��hypertension.��She��has��no��findings��consistent�
�with��a��diagnosis��of��preeclampsia,
although��she��is��now��at��risk��for��this��condition.��Current��evidence��suggest
s��that,��in��the��absence��of
preeclampsia��with��severe��features��(blood��pressure��>��160/110��mm��Hg,��thromb
ocytopenia,��impaired��liver
function,��renal��insufficiency,��pulmonary��edema,��cerebral��edema,��or��visual��
disturbance),��pregnancy��can
safely��be��continued��until��37��weeks��gestation.��At��that��point��delivery��is�
�recommended��in��order��to��avoid��the
risks��associated��with��gestational�hypertension,��which��include��abruption,��int
rauterine��growth��restriction,��and
progression��to��preeclampsia��and��eclampsia.

Item��199

This��patient��has��signs��and��symptoms��of��carpal��tunnel��syndrome.��Her��new��
task��is��likely��causing
inflammation��and/or��hypertrophy��of��the��flexor��tendons��of��the��wrist��and��h
and,��which��in��turn��are��placing
pressure��on��the��median��nerve��within��the��closed��space��of��the��carpal��tunn
el.��The��use��of��a��wrist��splint��and
NSAIDs��has��been��shown��to��reduce��pain��and��should��be��tried��along��with��ex
ercises��of��the��wrist��and��hand,��and
activity��modification��if��possible.��Nerve��conduction��studies��are��not��recomm
ended��unless��conservative
management��fails��and��carpal��tunnel��release��is��being��considered.

72
Item��200

The��term��Achilles��tendinopathy�reflects��the��chronic,��noninflammatory,��degene
rative��nature��of��this��disorder
as��opposed��to��the��traditional��term��tendinitis.��Eccentric��strengthening��pro
grams��of��the��gastrocnemius��and
soleus��muscles��have��demonstrated��60%-
90%��improvement��in��pain��and��function��and��are��considered��the
first-
line��treatment��for��chronic��midsubstance��Achilles��tendinopathy.��Oral��NSAIDs�
�and��oral��corticosteroids
are��ineffective,��and��local��corticosteroid��injection��in��or��near��a��load-
bearing��tendon��such��as��the��Achilles
tendon��increases��the��risk��of��tendon��rupture.��Surgical��tendinoplasty��is��in
consistently��beneficial��and��carries
added��risk,��and��is��not��a��first-line��treatment.

Item��201

Drug��therapy��aimed��at��increasing��HDL-
cholesterol��levels��when��added��to��a��statin��treatment��does��not
decrease��a��patient�s��cardiovascular��risk.��Such��agents��have��no��effect��on��
all-cause��mortality,��cardiovascular
mortality,��or��the��risk��of��stroke��(SOR��B).��Current��guidelines��for��treatme
nt��of��hyperlipidemia��are��based��on
individual��cardiovascular��risk��stratification��rather��than��LDL-
cholesterol��levels,��and��recommend��low-,
medium-,��or��high-
intensity��therapy��with��statins,��based��on��an��individual��patient�s��risk.

Item��202

Patients��with��diverticulosis��should��increase��dietary��fiber��intake��or��take�
�fiber��supplements��to��slow
progression��of��the��diverticular��disease.��Avoiding��nuts,��corn,��popcorn,��and
��small��seeds��has��not��been��shown
to��prevent��complications��of��diverticular��disease.��Limiting��intake��of��dairy
��products,��spicy��foods,��and��wheat
flour��would��be��appropriate��for��other��gastrointestinal��problems��such��as��la
ctose��intolerance,��gastroesophageal
reflux��disease��(GERD),��and��celiac��disease.

Item��203

Necrobiosis��lipoidica��diabeticorum��is��seen��in��0.3%��of��patients��with��diabe
tes��mellitus��or��impaired��glucose
tolerance.��The��lesions��may��precede��the��diagnosis��of��diabetes��mellitus��by�
�several��years.��The��sharply
demarcated��reddish-
brown��plaque��with��central��yellow��deposits��in��the��pretibial��area��is��chara
cteristic��and��a
biopsy��is��not��always��necessary.��Topical��corticosteroids��are��sometimes��help
ful.

73

Granuloma��annulare��and��sarcoidosis��are��unlikely��on��the��leg.��Early��lesions
��of��necrobiosis��lipoidica
diabeticorum��can��be��confused��with��granuloma��annulare��or��sarcoidosis,��howev
er,��and��a��biopsy��may��be
helpful.��While��xanthomas��can��be��flat��plaques��up��to��several��centimeters��i
n��size,��they��usually��occur��on��flexor
surfaces��of��the��limbs��along��with��the��trunk��and��face.��They��lack��the��red
dish-brown��outer��portion��of��the
lesions��described��here.

Item��204

Thiazolidinediones��(TZDs)��improve��hyperglycemia��by��improving��insulin��resista
nce��and��by��maintaining��or
improving��P-
cell��secretory��function.��One��of��the��side��effects��of��TZDs��is��that��they��
can��cause��fluid��retention,
especially��in��patients��with��cardiac��and��renal��disease.��This��may��lead��to�
�weight��gain��and��peripheral��edema.
Because��of��this��effect,��TZDs��are��contraindicated��in��patients��with��New��Yo
rk��Heart��Association��class��III��or
IV��heart��failure.��They��can��also��reduce��bone��mineral��density��and��are��ass
ociated��with��a��higher��risk��of
non-
osteoporotic��bone��fractures.��In��addition��to��their��ability��to��decrease��glu
cose��levels,��however,��there��have
been��some��other��favorable��effects��noted��with��their��use.��Clinical��trials��
have��shown��that��treatment��with
pioglitazone��resulted��in��a��significant��reduction��in��the��composite��outcome�
�of��nonfatal��acute��myocardial
infarction,��stroke,��and��all-
cause��mortality.��They��may��also��help��prevent��central��nervous��system��insuli
n
resistance-
related��cognitive��dysfunction.��TZDs��are��also��useful��in��patients��with��nona
lcoholic��steatohepatitis.

Item��205

Proton��pump��inhibitor��use��has��been��shown��to��increase��the��risk��for��Clost
ridium��difficile��and��other��enteric
infections,��and��elderly��patients��and��those��with��significant��comorbid��condi
tions��may��already��be��at��increased
risk.��Studies��have��not��shown��an��increased��risk��for��iron��deficiency.��Ther
e��is��no��increased��risk��for
hypothyroidism,��Helicobacterpylori��infection,��or��type��2��diabetes��mellitus.

74

Item��206

Nonbullous��impetigo��is��most��often��caused��by��Streptococcus��pyogenes��and��me
thicillin-sensitive
Staphylococcus��aureus.��Cephalexin��is��the��most��appropriate��option,��with��goo
d��coverage��for��both��of��these
bacteria.��Penicillin��VK��has��been��found��to��be��no��more��effective��than��pla
cebo��in��the��treatment��of��impetigo.
Macrolide��resistance��limits��the��use��of��erythromycin.��Tetracycline��should��n
ot��be��used��in��children��<��8��years
old��as��it��may��cause��staining��of�permanent��teeth.��Trimethoprim/sulfamethoxaz
ole��has��coverage��against��both
methicillin-sensitive��and��methicillin-
resistant��Staphylococcus�aureus,��but��may��have��inadequate��coverage��for
Streptococcus.

Item��207

Dopamine��antagonists,��such��as��metoclopramide,��block��dopamine��stimulation��in
��the��chemoreceptor��trigger
zone,��thereby��limiting��emetic��input��to��the��medullary��vomiting��center��(SOR
��C).��SSRIs,��such��as��ondansetron,
also��work��in��the��chemoreceptor��trigger��zone.��They��inhibit��serotonin��at��t
he��5-HT3��receptor��in��the��small
bowel,��vagus��nerve,��and��chemoreceptor��trigger��zone.��Antihistamines��and��ant
icholinergics��limit��stimulation
of��the��vomiting��center��through��inhibition��of��the��II,��receptor��and��acetyl
choline,��respectively.��These
medications��are��particularly��beneficial��in��vestibular-
mediated��nausea,��such��as��motion��sickness.

Item��208

Autosomal��dominant��polycystic��kidney��disease��(ADPCKD)��is��the��most��common��
genetic��kidney��disease��and
accounts��for��4.7%��of��end-
stage��kidney��disease��cases��in��America.��Many��patients��with��ADPCKD��are
asymptomatic,��but��early��symptoms��can��include��flank��pain,��gross��hematuria,�
�or��recurrent��urinary��tract
infections.��The��most��common��extrarenal��manifestation��of�ADPCKD��is��hypertens
ion,��which��can��precipitate
cardiovascular��dysfunction,��including��left��ventricular��hypertrophy.��Thus,��ea
rly��diagnosis��and��management
of��hypertension��is��crucial.

The��goal��blood��pressure��should��be��<��140/90��mm��Hg��in��patients��under��the
��age��of��60.��All��ADPCKD��patients
eventually��develop��a��loss��of��renal��function,��and��approximately��80%��develo
p��end-stage��renal��disease��by��age
70.��An��ACE��inhibitor��is��the��recommended��first-
line��therapy��(SOR��C),��so��lisinopril��is��the��best��choice��for
this��patient.��Angiotensin��receptor��blockers��are��acceptable��in��patients��who
��cannot��tolerate��ACE��inhibitors.

75

Item��209

Polymyalgia��rheumatica��(PMR)��without��concurrent��giant��cell��arteritis��is��tr
eated��with��a��slow��taper��of
low-dose��corticosteroids��(SOR��C).��Disease-
modifying��antirheumatic��drugs��and��antimalarial��medications
have��no��role��in��the��treatment��of��PMR.��Aspirin��therapy��is��recommended��as
��adjuvant��therapy��for��giant��cell
arteritis��to��decrease��stroke��risk��(SOR��C).��Colchicine��may��alleviate��acute
��symptoms��of�gout��and��pseudogout.

Item��210

Chronic��kidney��disease��appears��to��be��overdiagnosed��in��the��older��populatio
n.��Stage��3A��kidney��disease��is
defined��as��an��estimated��glomerular��filtration��rate��(GFR)��of��45-
59��mL/min/1.73��m2��and��is��predominantly
seen��in��older��patients.��It��is��seldom��progressive��in��the��absence��of��sign
ificant��proteinuria.��Older��patients��with
chronic��kidney��disease��are��less��likely��to��develop��end-
stage��renal��disease��than��to��die��of�complications��related
to��aging��and��cardiovascular��disease.��There��is��a��decline��in��estimated��GFR
��with��normal��aging,��and��the
likelihood��of��patients��progressing��to��end-
stage��renal��disease��and��dialysis��is��minimal��if��they��have��a��GFR��of
45-59��mL/min/1.73��m2.

Item��211

In��contrast��to��Little��League��elbow,��which��is��a��complex��of��possible��inju
ries,��Little��League��shoulder��refers
to��one��entity,��proximal��humeral��epiphysitis,��most��often��developing��as��an�
�overuse��injury��in��baseball��pitchers
age��11-16.

76

Item��212

The��most��common��cause��of��hypoglycemia��in��previously��stable,��well-
controlled��diabetic��patients��who��have
not��changed��their��diet��or��insulin��dosage��is��diabetic��renal��disease.��A��r
eduction��in��physical��activity��or��the
appearance��of��insulin��antibodies��(unlikely��after��20��years��of��therapy)��wou
ld��increase��insulin��requirements
and��produce��hyperglycemia.��Spontaneous��improvement��of�P-
cell��function��after��20��years��would��be��very��rare.

Item��213

Common causes of red eye include infectious conjunctivitis,


allergies, corneal abrasion,
keratoconjunctivitis,��subconjunctival��hemorrhage,��uveitis,��blepharitis,��iritis
,��acute��angle-closure��glaucoma,
and��herpes��zoster��ophthalmicus.

Viral��infections��typically��cause��conjunctivitis��with��mild��pain��and��no��los
s��of��vision.��The��problem��is��usually
unilateral��in��the��beginning��and��a��watery��to��serous��discharge��may��be��not
ed.��Adenovirus��is��the��most��common
cause.��Acute��bacterial��conjunctivitis��has��a��similar��presentation��and��may��
include��eyelid��edema��and��a��purulent
discharge.��Allergic��conjunctivitis��is��usually��bilateral��and��painless,��with�
�intense��itching,��and��a��stringy��or
ropy��watery��discharge.
Herpes��zoster��ophthalmicus��is��associated��with��a��vesicular��rash,��keratitis,
��and��uveitis.��The��rash��is��preceded
by��pain��and��a��tingling��sensation.��Findings��include��conjunctivitis��and��der
matomal��involvement,��which��are
usually��unilateral.

With��corneal��abrasion��there��is��usually��a��history��of��an��injury��involving�
�a��foreign��object.��Signs��and��symptoms
include��severe��eye��pain;��red,��watery��eyes;��photophobia;��and��a��foreign��bo
dy��sensation.��Vision��is��usually
normal��and��pupils��are��equal��and��reactive��to��light.

Symptoms��of��uveitis��include��a��red��eye,��loss��of��vision,��and��photophobia.�
�It��is��associated��with��many
autoimmune��diseases,��including��reactive��arthritis,��ankylosing��spondylitis,��a
nd��inflammatory��bowel��disease.

Acute��angle-
closure��glaucoma��causes��a��significant��loss��of��vision,��with��dilated��pupils
��that��don�t��react
normally��to��light.��Symptoms��include��severe��pain��and��watery��eyes,��with��ha
los��around��lights.��Patients��may
have��nausea��and��vomiting.��This��form��of��glaucoma��often��has��an��acute��onse
t.

77

Item��214

The��U.S.��Preventive��Services��Task��Force��recommends��against��the��use��of��P-
carotene��or��vitamin��E
supplementation��for��the��prevention��of��cardiovascular��disease��or��cancer.��Th
is��is��a��class��D��recommendation
(do��not��recommend).��Overall��there��is��no��beneficial��effect��on��cancer��or��
heart��disease��from��these��vitamin
supplements.��In��one��study��vitamin��E��appeared��to��increase��the��risk��of��he
morrhagic��stroke,��and��P-carotene
has��been��found��to��increase��the��risk��of��lung��cancer��in��persons��already��
at��higher��risk��for��lung��cancer.

Item��215

The��U.S.��Preventive��Services��Task��Force��recommends��that��primary��care��clin
icians��prescribe��oral��fluoride
supplementation��starting��at��6��months��of��age��for��children��whose��water��sup
ply��is��deficient��in��fluoride��(<��0.6
ppm)��(B��recommendation).��The��task��force��found��evidence��of��moderate��benefi
t��of��oral��fluoride
supplementation��for��the��prevention��of��dental��caries��in��this��group.

Item��216
For��all��infants��born��before��29��weeks��gestation,��palivizumab��is��recommende
d��for��the��first��year��of�life��during
respiratory��syncytial��virus��season��to��reduce��the��likelihood��of�hospitalizat
ion.��Immunization��against��pertussis
and��rotavirus��is��not��recommended��until��the��2-
month��visit.��Influenza��vaccine��is��not��recommended��for��any
infant��until��6��months��of��age.��There��is��no��indication��for��immunoglobulin�
�in��this��infant.

78

Item��217

The��treatment��goal��of��narrow��QRS��complex��tachycardia��is��to��slow��down��th
e��heart��rate��and��to��convert��to
normal��sinus��rhythm��by��blocking,��or��increasing��the��refractoriness��of,��the
��atrioventricular��node.��In��a
hemodynamically��stable��patient��vagal��maneuvers��are��a��good��first-
line��treatment,��followed��by��adenosine.
If�those��do��not��work��verapamil��or��diltiazem��can��be��used.��DC��cardioversio
n��is��used��in��narrow��QRS��complex
tachycardia��if��the��patient��becomes��hemodynamically��unstable.��Amiodarone,��pr
ocainamide,��and��sotalol��are
all��used��for��the��treatment��of��wide��QRS��complex��tachycardia��but��not��for�
�narrow-complex��tachycardia.

Item��218

This��patient��displays��most��of��the��criteria��for��borderline��personality��dis
order.��This��is��a��maladaptive
personality��type��that��is��present��from��a��young��age,��with��a��strong��geneti
c��predisposition.��It��is��estimated��to
be��present��in��1��
%��of��the��general��population��and��involves��equal��numbers��of��men��and��women
;��women��seek
care��more��often,��however,��leading��to��a��disproportionate��number��of��women��
being��identified��by��medical
providers.

Borderline��personality��disorder��is��defined��by��high��emotional��lability,��int
ense��anger,��unstable��relationships,
frantic��efforts��to��avoid��a��feeling��of��abandonment,��and��an��internal��sense
��of��emptiness.��Nearly��every��patient
with��this��disorder��engages��in��self-
injurious��behavior��(cutting,��suicidal��gestures��and��attempts),��and��about��1
in��10��patients��eventually��succeeds��in��committing��suicide.��However,��90�
%��of�patients��improve��despite��having
made��numerous��suicide��threats.��Suicidal��gestures��and��attempts��peak��when��p
atients��are��in��their��early��20s,
but��completed��suicide��is��most��common��after��age��30��and��usually��occurs��in
��patients��who��fail��to��recover��after
many��attempts��at��treatment.��In��contrast,��suicidal��actions��such��as��impulsi
ve��overdoses��or��superficial��cutting,
most��often��seen��in��younger��patients,��do��not��usually��carry��a��high��short-
term��risk,��and��serve��to��communicate
distress.

Inpatient��hospitalization��may��be��an��appropriate��treatment��option��if��the��p
erson��is��experiencing��extreme
difficulties��in��living��and��daily��functioning,��and��pharmacotherapy��may��offe
r��a��mild��degree��of��symptom
relief.��While��these��modalities��have��a��role��in��certain��patients,��psychothe
rapy��is��considered��the��mainstay��of
therapy,��especially��in��a��relatively��stable��patient��such��as��the��one��descr
ibed.

79

Item��219

Absorption��and��metabolism��of��vitamin��D��is��known��to��be��affected��by��inter
action��with��other��medications.
Isoniazid��and��thiazide��diuretics��can��lead��to��increased��blood��levels��or��a
ctivity��of��vitamin��D.��Estrogen
replacement��therapy��can��also��increase��levels��of�vitamin��D��in��the��blood,��
although��this��potential��benefit��seems
to��diminish��when��progesterone��is��added.��Vitamin��D��absorption��through��the�
�gut��can��be��reduced��by��mineral
oil,��cholestyramine,��and��certain��antacid��preparations,��leading��to��lower��bl
ood��levels.��The��metabolism��of
vitamin��D��is��accelerated��by��anticonvulsant��drugs��such��as��phenobarbital��an
d��phenytoin,��which��can��also��result
in��lower��than��desired��levels��of��vitamin��D.��Statins��are��not��reported��to�
�have��any��known��effect��on��vitamin��D
levels.

Item��220

This��case��presents��a��child��with��an��insidious��onset��of��pain��and��mild��li
mitation��of��range��of��motion��of��one��hip
joint.��The��case��suggests��involvement��of��the��femoroacetabular��joint,��and��a
ll��of��the��listed��options��can��affect
this��joint.��Idiopathic��osteonecrosis��of��the��hip��(Legg-Calve-
Perthes��disease)��occurs��most��commonly��in
children��at��2-
12��years��of��age��and��has��a��male��predominance.��Symptom��onset��is��insidious
,��as��in��this��case,
with��symptom��severity��and��functional��limitations��dependent��on��the��level��o
f��disease��progression.��Slipped
capital��femoral��epiphysis��occurs��much��more��commonly��in��adolescents.��Transi
ent��synovitis��and��septic
arthritis��have��a��more��acute��onset,��and��typically��cause��fever.��Osteoarthri
tis��typically��occurs��in��older��adults.

Item��221

A��number��of��alternative��therapies��have��been��used��for��problems��related��to
��pregnancy,��although��vigorous
studies��are��not��always��possible.��For��nausea��and��vomiting,��however,��vitami
n��B6��is��considered��first-line
therapy,��sometimes��combined��with��doxylamine.��Other��measures��that��have��been
��found��to��be��somewhat
useful��include��ginger��and��acupressure.

Cranberry��products��can��be��useful��for��preventing��urinary��tract��infections,�
�and��could��be��recommended��for
patients��if��this��is��a��concern.��Blue��cohosh��has��been��used��as��a��partus��
preparator,��but��there��are��concerns��about
its��safety.��Fenugreek��has��been��used��to��increase��milk��production��in��breas
tfeeding��mothers,��but��no��rigorous
trials��have��been��performed.
80

Item��222

Most��of��the��symptoms��of��foodborne��illness��are��not��specific��to��the��causa
tive��organism.��However,��the��onset
of��vomiting��and��diarrhea��within��hours��of��consumption��of��contaminated��food
��results��from��the��ingestion��of
preformed��toxins,��most��often��from��Staphylococcus��aureus��or��Bacillus��cereus
��in��the��United��States.
Campylobacter�jejuni,��Escherichia��coli��0157:117,��Shigella��species,��and��hepat
itis��A��typically��produce
symptoms��more��than��a��day��after��ingestion.

Item��223

For��functional��constipation��in��older��adults,��behavioral��changes��should��be�
�first-line��management.��These
include��scheduled��toileting��with��proper��positioning,��increased��intake��of��f
iber��and��fluids,��and��avoiding
bedpan��use.��If��there��is��not��an��adequate��response��to��behavioral��intervent
ions,��osmotic��laxatives��should��be
initiated.��Polyethylene��glycol��is��more��effective��and��has��fewer��side��effec
ts��compared��to��lactulose.
Magnesium��salts,��including��magnesium��citrate,��do��not��have��strong��evidence�
�for��safety��or��efficacy,��and
magnesium��toxicity��is��a��concern��with��long-
term��use.��Due��to��possible��adverse��effects��of��stimulant��laxatives
in��the��long��term,��particularly��with��older��adults,��these��drugs��should��be�
�used��only��if��fiber��and��osmotic
laxatives��are��unsuccessful.

Item��224

SSRIs��and��SNRIs��are��both��effective��in��reducing��depressive��symptoms,��but��
SNRIs��have��been��shown��to��be
superior��to��SSRIs��for��management��of��neuropathic��pain��(SOR��A).��Bupropion��
would��effectively��treat��the
patient�s��depression��and��could��cause��weight��loss,��but��it��is��contraindicat
ed��in��patients��with��seizure��disorders
(SOR��A).��Tricyclic��antidepressants��such��as��nortriptyline��could��also��help��
with��the��pain��but��might��also��worsen
the��patient�s��obesity��and��fatigue��(SOR��A).

81

Item��225

An��ulnar��neuropathy��most��commonly��presents��with��sensory��changes��in��the��f
ourth��and��fifth��digits��and��usually
does��not��involve��weakness��in��hand��grip.��There��is��usually��no��specific��in
jury,��but��any��activity��that��results��in
repetitive��or��prolonged��wrist��extension,��as��with��cycling��or��playing��catch
er,��may��increase��the��risk��of��this
problem.��It��is��important��to��examine��the��neck��for��cervical��disc��disease��
and��to��examine��the��shoulder��to��see
if��motion��elicits��the��pain,��which��would��indicate��a��brachial��plexus��probl
em.��If��symptoms��are��reproduced��by
compressing��the��ulnar��nerve��at��the��elbow��this��could��be��the��site��of��ent
rapment.��Clinical��tests��may��include
a��positive��Tinel��sign��on��percussion��of��the��ulnar��nerve��over��Guyon�s��can
al.��Also,��there��may��be��a��positive
Phalen��sign,��with��maximum��passive��flexion��of��the��wrist��for��1��minute��ind
ucing��paresthesias��in��the��fourth
and��fifth��fingers.

With�ulnar��neuropathy,��plain��radiographs��are��usually��normal.��Ultrasonography
��of�the��peripheral��nerves��may
be��helpful��in��identifying��compression��etiologies.��However,��electromyelograph
y��and��nerve��conduction
velocities��may��be��required��to��identify��the��area��of��entrapment.

Item��226
Self-
measured��blood��pressure��monitoring,��with��or��without��additional��support��(e.
g.,��education,��counseling,
telemedicine,��home��visits,��Web-
based��logging),��lowers��blood��pressure��when��compared��with��usual��care,
although��the��benefits��beyond��12��months��are��not��clear��(SOR��A).��Limiting��
sodium��intake��to��2400��mg/day
(approximately��1��teaspoon��of�table��salt)��is��recommended��to��lower��blood��pr
essure.��Additional��benefit��occurs
with��a��limit��of��1500��mg/day��(SOR��B).��A��diet��that��emphasizes��vegetables,
��fruits,��and��whole��grains��is��also
recommended��to��lower��blood��pressure��(SOR��A),��as��well��as��limiting��alcohol
��consumption��to��no��more��than
2��drinks/day��for��men,��and��1��drink/day��for��women��(SOR��C).��Because��of��mi
xed��results��from��therapeutic
trials��and��many��limitations��to��the��trials,��the��American��Heart��Association
��does��not��recommend��either��yoga
or��acupuncture��to��lower��blood��pressure.

82

Item��227

Daytime��sleepiness��is��the��clinically��relevant��symptom��of��obstructive��sleep
��apnea��(OSA)��that��is��most
responsive��to��treatment.��Other��associated��symptoms��such��as��snoring,��insomn
ia,��and��fatigue��are��either��less
clinically��relevant��or��less��responsive��to��treatment.��Hypertension,��diabetes
��mellitus,��and��coronary��artery
disease��are��associated��with��OSA,��but��evidence��is��insufficient��that��contin
uous��positive��airway��pressure
(CPAP)��improves��outcomes��for��these��conditions,��especially��if��they��are��not
��associated��with��daytime
sleepiness.

Item��228

A��baseline��EKG��is��currently��not��recommended��in��an��asymptomatic��patient��t
o��screen��for��cardiovascular
disease.��The��American��Academy��of��Family��Physicians��(AAFP)��specifically��adv
ises��against��performing��a
baseline��EKG.
The��AAFP��does��endorse��performing��a��detailed��past��medical��history��to��excl
ude��a��history��of��hypertension,
chest��pain,��prior��heart��murmur,��or��syncope.��It��also��recommends��obtaining�
�a��family��history��that��includes
questions��about��any��family��members��with��prolonged��QT��syndrome,��Marfan��syn
drome,��or��sudden��death
before��the��age��of��50.��A��physical��examination��is��recommended,��including��p
alpation��of��the��femoral��pulses
simultaneously��to��detect��coarctation��of��the��aorta,��as��well��as��heart��ausc
ultation��performed��with��the��patient
both��supine��and��standing,��and��with��the��Valsalva��maneuver,��to��detect��a��h
eart��murmur��suggestive��of��cardiac
disease.

Item��229

This��patient��has��transient��tachypnea��of��the��newborn,��which��typically��occu
rs��within��2��hours��of��birth.��The
chest��radiograph��usually��shows��hyperexpansion��with��perihilar��densities��and�
�fluid��within��the��fissures.
Respiratory��distress��syndrome��of��the��newborn��is��most��often��seen��in��prema
ture��infants,��and��the��chest
radiograph��shows��a��classic��diffuse��ground-
glass��appearance.��With��pneumothorax��a��chest��radiograph��would
typically��show��a��partial��or��complete��lung��collapse.��Meconium��aspiration��s
yndrome��occurs��in��the��setting��of
meconium-
stained��fluid��and��is��usually��apparent��immediately��after��delivery.��The��che
st��radiograph��typically
shows��fluffy��densities��with��hyperinflation.

83

Item��230

In��critically��ill��adult��patients,��the��only��deep��vein��thrombosis��prophylax
is��that��decreases��mortality��risk��is
anticoagulation��therapy.��Mechanical��device��prophylaxis��does��not��lower��the��
mortality��risk��compared��to��no
prophylaxis.��Data��suggests��that��patients��managed��with��both��prophylactic��an
ticoagulation��and��mechanical
device��prophylaxis��have��a��higher��mortality��risk��than��those��managed��by��pr
ophylactic��anticoagulation��alone.

Item��231

Panic��disorder��typically��presents��in��late��adolescence��or��early��adulthood��
with��unpredictable��episodes��of
palpitations,��sweating,��gastrointestinal��distress,��dizziness,��and��paresthesia
s.��The��attacks��are��sporadic��and
last��10-
60��minutes.��Generalized��anxiety��disorder��is��more��common,��and��common��sympt
oms��include
restlessness,��fatigue,��muscle��tension,��irritability,��difficulty��concentrating
,��and��sleep��disturbance.
Pheochromocytoma��is��associated��with��headache��and��hypertension,��and��usually�
�occurs��in��thin��patients.
Paroxysmal��supraventricular��tachycardia��is��usually��not��associated��with��gast
rointestinal��distress��or
paresthesias.��While��mitral��valve��prolapse��can��be��associated��with��anxiety��
and��panic��disorder,��the��physical
examination��would��not��be��normal.

Item��232

The��U.S.��Preventive��Services��Task��Force��recommends��one-
time��conventional��abdominal��duplex
ultrasonography��for��screening��patients��who��are��at��risk��of��abdominal��aorti
c��aneurysm��(males��65-75��years
of��age��who��have��smoked��a��total��of��100��cigarettes��or��more��during��their�
�lifetime).��This��imaging��modality��has
high��sensitivity��and��specificity,��and��it��is��noninvasive,��easy��to��use,��an
d��low��cost.��None��of��the��other��imaging
modalities��have��been��formally��evaluated��in��clinical��trials.

84

Item��233

Repetitive��high-
intensity��training��places��an��individual��at��risk��for��developing��a��stress��
fracture.��A��plain
radiograph��is��the��best��initial��test��for��a��suspected��stress��fracture.��If�
�this��is��negative,��then��the��study��should
be��repeated��in��2-
3��weeks.��MRI��is��now��considered��the��procedure��of��choice��if��there��is��an�
�urgent��need��for��a
diagnosis,��although��triple-
phase��bone��scintigraphy��has��a��similar��sensitivity.

Treatment��of��stress��fractures��usually��consists��of��decreasing��activity��or,�
�in��some��instances,��such��as
involvement��of��the��anterior��tibial��cortex��(where��there��is��a��risk��of��a��
complete��fracture),��non-weight��bearing
with��immobilization.��Using��a��walker��boot��for��tibial��stress��fractures��redu
ces��the��time��to��resumption��of��full
activity.��Most��stress��fractures��should��not��be��treated��with��a��bone��stimul
ator.��Supplementation��with��vitamin
D��and��calcium��has��shown��some��benefit��in��prevention,��whereas��bisphosphonat
es��have��not.��NSAIDs��are
relatively��contraindicated,��as��limited��studies��have��shown��that��they��may��a
ctually��inhibit��healing��of��traumatic
fractures.
Item��234

Up��to��28��
%��of��patients��may��be��defined��as��having��resistant��hypertension��(not��contr
olled��on��three��drugs��or
controlled��on��four��or��more��drugs).��Primary��aldosteronism��is��present��in��u
p��to��5%-10%��of��all��hypertensive
patients��and��7�%�-20�
%��of�those��with��resistant��hypertension.��This��may��be��due��to��bilateral��adr
enal��hyperplasia
or��a��unilateral��aldosterone-
secreting��adenoma,��which��can��be��diagnosed��if��there��is��elevated��serum
aldosterone��in��the��presence��of��suppressed��renin��levels.

A��cortisol��level��and��a��dexamethasone��suppression��test��are��appropriate��tes
ts��for��Cushing��syndrome.��A
17-
hydroxyprogesterone��level��tests��for��congenital��adrenal��hyperplasia.��Renal��u
ltrasonography��will��not
adequately��screen��for��any��of��these��conditions.

Item��235

This��patient��has��molluscum��contagiosum,��which��can��be��easily��treated��by��c
uretting��the��lesions��and��inducing
an��inflammatory��reaction.��The��lesions��may��disappear��spontaneously��in��a��fe
w��months;��however,��the��best
management��of��this��condition��in��a��patient��who��is��bothered��by��it��is��to�
�induce��resolution.
85
Item��236

The��patient��has��a��mallet��fracture,��an��avulsion��fracture��of��the��distal��p
halanx��with��a��bone��fragment��on��the
terminal��extensor��tendon,��resulting��in��unopposed��flexion��and��the��inability
��to��actively��extend��the��distal
interphalangeal��(DIP)�joint.��Conservative��treatment��consists��of�immobilization
��of�the��DIP�joint��in��extension
for��8��weeks��and��is��recommended��for��most��cases��of��mallet��fracture.��If��t
he��joint��is��allowed��to��flex��at��any��time
during��that��period,��then��the��treatment��period��must��be��extended.��Immobiliz
ation��of��the��proximal
interphalangeal��(PIP)��joint��is��not��required.��Surgical��pinning��may��be��indi
cated��in��more��complicated��fractures
or��with��failure��of��conservative��therapy.

Item 237

Pulmonary��rehabilitation��should��be��considered��inpatients��with��COPD��who��are
��optimally��medically��managed
and��continue��to��have��symptoms,��particularly��dyspnea.��Pulmonary��rehabilitati
on��improves��exercise��capacity,
dyspnea,��and��health-
related��quality��of��life��outcomes��in��patients��with��COPD.��Supplemental��oxyg
en��use��has
been��shown��to��decrease��mortality��in��patients��with��COPD��who��have��severe��
hypoxemia.��Short-acting
anticholinergics��such��as��ipratropium��should��be��used��for��patients��with��mil
d��disease��requiring��only��as-needed
medications.��The��long-
acting��anticholinergic��tiotropium��has��been��shown��to��improve��quality-of-
life��scores.

Item��238

Nonverbal��communication��is��important��for��identifying��issues��that��a��patient
��may��be��hiding��or��be��unwilling
to��divulge.��Some��nonverbal��clues,��however,��are��culturally��based.��Many��old
er��or��less-educated
Mexican-
Americans��consider��direct��eye��contact��to��be��disrespectful.��Because��a��phys
ician��is��held��in��high
regard,��these��patients��will��often��either��look��down��or��look��at��another,��
more���equal���person��in��the��room
while��being��interviewed.��Many��Americans,��on��the��other��hand,��may��consider�
�a��lack��of��eye��contact��to��be
negative,��and��that��it��indicates��that��a��patient��is��unsure��of��the��informa
tion��they��are��providing,��has��poor
self-esteem,��or��is��hiding��something.

86

Item��239

In��a��female��>35��years��old,��smoking��15��or��more��cigarettes��per��day��poses
��an��unacceptable��health��risk��with
the��use��of��combined��oral��contraceptives.��Patients��with��varicose��veins��are
��not��at��increased��risk��for��deep��vein
thrombosis��(DVT)/pulmonary��embolism,��which��would��be��an��unacceptable��health�
�risk��with��the��use��of
combined��oral��contraceptives.��Combined��oral��contraceptives��actually��lower��t
he��risk��of��ovarian��cancer,��and
women��may��continue��to��use��them��while��awaiting��treatment��for��ovarian��canc
er.��Women��with��a��BMI��>30.0
kg/m2��who��take��oral��contraceptives��are��more��likely��to��develop��DVTs��than�
�those��who��do��not��use��them,��but
the��advantages��of��oral��contraceptives��are��considered��to��be��greater��than��
the��disadvantages��in��these��patients,
and��obesity��is��not��an��absolute��contraindication.��A��patient��with��sickle��c
ell��disease��is��at��a��higher��risk��of
adverse��events��from��an��unintended��pregnancy��than��from��the��use��of��combine
d��oral��contraceptives.

Item��240
An��understanding��of��the��anatomy��and��function��of��the��components��of��the��k
nee,��coupled��with��a��clear
description��of��the��traumatic��event,��is��essential��for��making��an��accurate��
initial��clinical��assessment��of
sports-
related��knee��injuries.��The��posterior��cruciate��ligament��(PCL)��connects��the�
�medial��femoral��condyle
to��the��posterior��intercondylar��area��of��the��tibia��and��is��affixed��in��such
��a��way��that��the��anterolateral��section��is
taut��in��flexion��and��the��posteromedial��section��is��taut��in��extension,��help
ing��to��maintain��the��correct��anatomic
relationship��between��the��femur��and��tibia.��The��PCL��alone��provides��almost��
all��of��the��resistance��to��posterior
displacement��of��the��tibia��and��so��is��appropriately��the��strongest��of��the��
cruciate��ligaments.��Tearing��or��rupture
of��the��PCL��can��occur��with��hyperextension,��hyperflexion,��or��rotation��appli
ed��with��a��force��that��is��so��great
that��other��knee��components��are��also��generally��injured.

The��most��common��mechanism��leading��to��an��isolated��injury��of��the��PCL��is��
a��direct��blow��to��the��anterior��tibia
with��the��knee��in��flexion,��like��that��experienced��when��the��proximal��tibia�
�impacts��the��dashboard��in��an
automobile��crash��or��when��an��athlete��is��hit��or��kicked��in��the��proximal��t
ibia��while��the��knee��is��in��flexion.
Athletes��with��a��PCL��injury��frequently��complain��of��posterior��knee��pain��an
d��pain��when��kneeling.��The
presence��of��painful��limitation��of��flexion��and��a��posterior��sag��sign��(post
erior��drawer��sign)��on��examination
strongly��supports��a��diagnosis��of��isolated��PCL��injury.

87

You might also like